RN PEDIA

June 1, 2016 | Author: Sarah Matilda Saunders Fitzgerald | Category: N/A
Share Embed Donate


Short Description

Download RN PEDIA...

Description

Your Report Correct Q.1)

66% answered this correctly. 34% answered wrong

Which of the following constitutes a break in sterile technique while preparing a sterile field for a dressing change? A. Using sterile forceps, rather than sterile gloves, to handle a sterile item B. Touching the outside wrapper of sterilized material without sterile gloves C. Placing a sterile object on the edge of the sterile field (your answer) D. Pouring out a small amount of solution (15 to 30 ml) before pouring the solution into a sterile container Explanation The edges of a sterile field are considered contaminated. When sterile items are allowed to come in contact with the edges of the field, the sterile items also become contaminated.

Correct Q.2)

86% answered this correctly. 14% answered wrong

The nurse explains to a patient that a cough: A. Is a protective response to clear the respiratory tract of irritants(your answer) B. Is primarily a voluntary action C. Is induced by the administration of an antitussive drug D. Can be inhibited by “splinting” the abdomen Explanation Coughing, a protective response that clears the respiratory tract of irritants, usually is involuntary; however it can be voluntary, as when a patient is taught to perform coughing exercises. An antitussive drug inhibits coughing. Splinting the abdomen supports the abdominal muscles when a patient coughs.

Correct Q.3)

73% answered this correctly. 27% answered wrong

The correct method for determining the vastus lateralis site for I.M. injection is to: A. Locate the upper aspect of the upper outer quadrant of the buttock about 5 to 8 cm below the iliac crest B. Palpate the lower edge of the acromion process and the midpoint lateral aspect of the arm C. Palpate a 1” circular area anterior to the umbilicus Divide the area between the greater femoral trochanter and the lateral femoral condyle into thirds, and select the middle third D. on the anterior of the thigh (your answer) Explanation The vastus lateralis, a long, thick muscle that extends the full length of the thigh, is viewed by many clinicians as the site of choice for I.M. injections because it has relatively few major nerves and blood vessels. The middle third of the muscle is recommended as the injection site. The patient can be in a supine or sitting position for an injection into this site.

Correct Q.4)

93% answered this correctly. 7% answered wrong

The most appropriate time for the nurse to obtain a sputum specimen for culture is: A. Early in the morning (your answer) B. After the patient eats a light breakfast C. After aerosol therapy D. After chest physiotherapy Explanation Obtaining a sputum specimen early in this morning ensures an adequate supply of bacteria for culturing and decreases the risk of contamination from food or medication.

Incorrect Q.5)

56% answered this correctly. 44% answered wrong

Which of the following will probably result in a break in sterile technique for respiratory isolation? A. Opening the patient‟s window to the outside environment (your answer) B. Turning on the patient‟s room ventilator C. Opening the door of the patient‟s room leading into the hospital corridor (correct answer) D. Failing to wear gloves when administering a bed bath Explanation Respiratory isolation, like strict isolation, requires that the door to the door patient‟s room remain closed. However, the patient‟s room should be well ventilated, so opening the window or turning on the ventricular is desirable. The nurse does not need to wear gloves for respiratory isolation, but good hand washing is important for all types of isolation.

Incorrect

74% answered this correctly. 26% answered wrong

Q.6)

A clinical nurse specialist is a nurse who has: A. Been certified by the National League for Nursing B. Received credentials from the Philippine Nurses‟ Association C. Graduated from an associate degree program and is a registered professional nurse (your answer) D. Completed a master‟s degree in the prescribed clinical area and is a registered professional nurse. (correct answer) Explanation A clinical nurse specialist must have completed a master‟s degree in a clinical specialty and be a registered professional nurse. The National League of Nursing accredits educational programs in nursing and provides a testing service to evaluate student nursing competence but it does not certify nurses. The American Nurses Association identifies requirements for certification and offers examinations for certification in many areas of nursing., such as medical surgical nursing. These certification (credentialing) demonstrates that the nurse has the knowledge and the ability to provide high quality nursing care in the area of her certification. A graduate of an associate degree program is not a clinical nurse specialist: however, she is prepared to provide bed side nursing with a high degree of knowledge and skill. She must successfully complete the licensing examination to become a registered professional nurse.

Correct Q.7)

64% answered this correctly. 36% answered wrong

The purpose of increasing urine acidity through dietary means is to: A. Decrease burning sensations B. Change the urine‟s color C. Change the urine‟s concentration D. Inhibit the growth of microorganisms (your answer) Explanation Microorganisms usually do not grow in an acidic environment.

Correct Q.8)

67% answered this correctly. 33% answered wrong

The ELISA test is used to: A. Screen blood donors for antibodies to human immunodeficiency virus (HIV) B. Test blood to be used for transfusion for HIV antibodies C. Aid in diagnosing a patient with AIDS D. All of the above (your answer) Explanation The ELISA test of venous blood is used to assess blood and potential blood donors to human immunodeficiency virus (HIV). A positive ELISA test combined with various signs and symptoms helps to diagnose acquired immunodeficiency syndrome (AIDS)

Correct Q.9)

61% answered this correctly. 39% answered wrong

Which element in the circular chain of infection can be eliminated by preserving skin integrity? A. Host B. Reservoir C. Mode of transmission D. Portal of entry (your answer) Explanation In the circular chain of infection, pathogens must be able to leave their reservoir and be transmitted to a susceptible host through a portal of entry, such as broken skin.

Correct Q.10)

73% answered this correctly. 27% answered wrong

Immobility impairs bladder elimination, resulting in such disorders as A. Increased urine acidity and relaxation of the perineal muscles, causing incontinence B. Urine retention, bladder distention, and infection (your answer) C. Diuresis, natriuresis, and decreased urine specific gravity D. Decreased calcium and phosphate levels in the urine Explanation The immobilized patient commonly suffers from urine retention caused by decreased muscle tone in the perineum. This leads to bladder distention and urine stagnation, which provide an excellent medium for bacterial growth leading to infection. Immobility also results in more alkaline urine with excessive amounts of calcium, sodium and phosphate,a gradual decrease in urine production, and an increased specific gravity.

Incorrect Q.11)

The two blood vessels most commonly used for TPN infusion are the:

17% answered this correctly. 83% answered wrong

A. Subclavian and jugular veins B. Brachial and subclavian veins (your answer) C. Femoral and subclavian veins D. Brachial and femoral veins (correct answer) Explanation Tachypnea (an abnormally rapid rate of breathing) would indicate that the patient was still hypoxic (deficient in oxygen).The partial pressures of arterial oxygen and carbon dioxide listed are within the normal range. Eupnea refers to normal respiration. Correct Q.12)

82% answered this correctly. 18% answered wrong

A patient who develops hives after receiving an antibiotic is exhibiting drug: A. Tolerance B. Idiosyncrasy C. Synergism D. Allergy (your answer) Explanation A drug-allergy is an adverse reaction resulting from an immunologic response following a previous sensitizing exposure to the drug. The reaction can range from a rash or hives to anaphylactic shock. Tolerance to a drug means that the patient experiences a decreasing physiologic response to repeated administration of the drug in the same dosage. Idiosyncrasy is an individual‟s unique hypersensitivity to a drug, food, or other substance; it appears to be genetically determined. Synergism, is a drug interaction in which the sum of the drug‟s combined effects is greater than that of their separate effects.

Correct Q.13)

49% answered this correctly. 51% answered wrong

Effective hand washing requires the use of: A. Soap or detergent to promote emulsification (your answer) B. Hot water to destroy bacteria C. A disinfectant to increase surface tension D. All of the above Explanation Soaps and detergents are used to help remove bacteria because of their ability to lower the surface tension of water and act as emulsifying agents. Hot water may lead to skin irritation or burns.

Correct Q.14)

47% answered this correctly. 53% answered wrong

The appropriate needle gauge for intradermal injection is: A. 20G B. 22G C. 25G D. 26G (your answer) Explanation Because an intradermal injection does not penetrate deeply into the skin, a small-bore 25G needle is recommended. This type of injection is used primarily to administer antigens to evaluate reactions for allergy or sensitivity studies. A 20G needle is usually used for I.M. injections of oilbased medications; a 22G needle for I.M. injections; and a 25G needle, for I.M. injections; and a 25G needle, for subcutaneous insulin injections.

Correct Q.15)

77% answered this correctly. 23% answered wrong

Which of the following conditions may require fluid restriction? A. Fever B. Chronic Obstructive Pulmonary Disease C. Renal Failure (your answer) D. Dehydration Explanation In real failure, the kidney loses their ability to effectively eliminate wastes and fluids. Because of this, limiting the patient‟s intake of oral and I.V. fluids may be necessary. Fever, chronic obstructive pulmonary disease, and dehydration are conditions for which fluids should be encouraged.

Correct Q.16)

68% answered this correctly. 32% answered wrong

The physician orders an IV solution of dextrose 5% in water at 100ml/hour. What would the flow rate be if the drop factor is 15 gtt = 1 ml?

A. 5 gtt/minute B. 13 gtt/minute C. 25 gtt/minute (your answer) D. 50 gtt/minute Explanation 100ml/60 min X 15 gtt/ 1 ml = 25 gtt/minute Correct Q.17)

83% answered this correctly. 17% answered wrong

A patient with no known allergies is to receive penicillin every 6 hours. When administering the medication, the nurse observes a fine rash on the patient‟s skin. The most appropriate nursing action would be to: A. Withhold the moderation and notify the physician (your answer) B. Administer the medication and notify the physician C. Administer the medication with an antihistamine D. Apply corn starch soaks to the rash Explanation Initial sensitivity to penicillin is commonly manifested by a skin rash, even in individuals who have not been allergic to it previously. Because of the danger of anaphylactic shock, he nurse should withhold the drug and notify the physician, who may choose to substitute another drug. Administering an antihistamine is a dependent nursing intervention that requires a written physician‟s order. Although applying corn starch to the rash may relieve discomfort, it is not the nurse‟s top priority in such a potentially life-threatening situation.

Correct Q.18)

38% answered this correctly. 62% answered wrong

Effective skin disinfection before a surgical procedure includes which of the following methods? A. Shaving the site on the day before surgery B. Applying a topical antiseptic to the skin on the evening before surgery C. Having the patient take a tub bath on the morning of surgery D. Having the patient shower with an antiseptic soap on the evening before and the morning of surgery (your answer) Explanation Studies have shown that showering with an antiseptic soap before surgery is the most effective method of removing microorganisms from the skin. Shaving the site of the intended surgery might cause breaks in the skin, thereby increasing the risk of infection; however, if indicated, shaving, should be done immediately before surgery, not the day before. A topical antiseptic would not remove microorganisms and would be beneficial only after proper cleaning and rinsing. Tub bathing might transfer organisms to another body site rather than rinse them away.

Correct Q.19)

55% answered this correctly. 45% answered wrong

All of the following statement are true about donning sterile gloves except: A. The first glove should be picked up by grasping the inside of the cuff. B. The second glove should be picked up by inserting the gloved fingers under the cuff outside the glove. C. The gloves should be adjusted by sliding the gloved fingers under the sterile cuff and pulling the glove over the wrist D. The inside of the glove is considered sterile (your answer) Explanation The inside of the glove is always considered to be clean, but not sterile.

Correct Q.20)

76% answered this correctly. 24% answered wrong

An infected patient has chills and begins shivering. The best nursing intervention is to: A. Apply iced alcohol sponges B. Provide increased cool liquids C. Provide additional bedclothes (your answer) D. Provide increased ventilation Explanation In an infected patient, shivering results from the body‟s attempt to increase heat production and the production of neutrophils and phagocytotic action through increased skeletal muscle tension and contractions. Initial vasoconstriction may cause skin to feel cold to the touch. Applying additional bed clothes helps to equalize the body temperature and stop the chills. Attempts to cool the body result in further shivering, increased metabloism, and thus increased heat production.

Incorrect Q.21)

66% answered this correctly. 34% answered wrong

Which of the following nursing interventions is considered the most effective form or universal precautions?

A. Cap all used needles before removing them from their syringes B. Discard all used uncapped needles and syringes in an impenetrable protective container (correct answer) C. Wear gloves when administering IM injections (your answer) D. Follow enteric precautions Explanation According to the Centers for Disease Control (CDC), blood-to-blood contact occurs most commonly when a health care worker attempts to cap a used needle. Therefore, used needles should never be recapped; instead they should be inserted in a specially designed puncture resistant, labeled container. Wearing gloves is not always necessary when administering an I.M. injection. Enteric precautions prevent the transfer of pathogens via feces. 71% answered this correctly. 29% answered wrong

Correct A natural body defense that plays an active role in preventing infection is:

Q.22)

A. Yawning B. Body hair (your answer) C. Hiccupping D. Rapid eye movements Explanation Hair on or within body areas, such as the nose, traps and holds particles that contain microorganisms. Yawning and hiccupping do not prevent microorganisms from entering or leaving the body. Rapid eye movement marks the stage of sleep during which dreaming occurs. Incorrect Q.23)

80% answered this correctly. 20% answered wrong

After 5 days of diuretic therapy with 20mg of furosemide (Lasix) daily, a patient begins to exhibit fatigue, muscle cramping and muscle weakness. These symptoms probably indicate that the patient is experiencing: A. Hypokalemia (correct answer) B. Hyperkalemia (your answer) C. Anorexia D. Dysphagia Explanation Fatigue, muscle cramping, and muscle weaknesses are symptoms of hypokalemia (an inadequate potassium level), which is a potential side effect of diuretic therapy. The physician usually orders supplemental potassium to prevent hypokalemia in patients receiving diuretics. Anorexia is another symptom of hypokalemia. Dysphagia means difficulty swallowing.

Correct Q.24)

65% answered this correctly. 35% answered wrong

Which of the following statements about chest X-ray is false? . A. No contradictions exist for this test (your answer) B. Before the procedure, the patient should remove all jewelry, metallic objects, and buttons above the waist C. A signed consent is not required D. Eating, drinking, and medications are allowed before this test Explanation Pregnancy or suspected pregnancy is the only contraindication for a chest X-ray. However, if a chest X-ray is necessary, the patient can wear a lead apron to protect the pelvic region from radiation. Jewelry, metallic objects, and buttons would interfere with the X-ray and thus should not be worn above the waist. A signed consent is not required because a chest X-ray is not an invasive examination. Eating, drinking and medications are allowed because the X-ray is of the chest, not the abdominal region.

Correct Q.25)

69% answered this correctly. 31% answered wrong

Thrombophlebitis typically develops in patients with which of the following conditions? A. Increases partial thromboplastin time B. Acute pulsus paradoxus C. An impaired or traumatized blood vessel wall (your answer) D. Chronic Obstructive Pulmonary Disease (COPD) Explanation The factors, known as Virchow‟s triad, collectively predispose a patient to thromboplebitis; impaired venous return to the heart, blood hypercoagulability, and injury to a blood vessel wall. Increased partial thromboplastin time indicates a prolonged bleeding time during fibrin clot formation, commonly the result of anticoagulant (heparin) therapy. Arterial blood disorders (such as pulsus paradoxus) and lung diseases (such as COPD) do not necessarily impede venous return of injure vessel walls.

Correct

81% answered this correctly. 19% answered wrong

Q.26)

Sterile technique is used whenever: A. Strict isolation is required B. Terminal disinfection is performed C. Invasive procedures are performed (your answer) D. Protective isolation is necessary Explanation All invasive procedures, including surgery, catheter insertion, and administration of parenteral therapy, require sterile technique to maintain a sterile environment. All equipment must be sterile, and the nurse and the physician must wear sterile gloves and maintain surgical asepsis. In the operating room, the nurse and physician are required to wear sterile gowns, gloves, masks, hair covers, and shoe covers for all invasive procedures. Strict isolation requires the use of clean gloves, masks, gowns and equipment to prevent the transmission of highly communicable diseases by contact or by airborne routes. Terminal disinfection is the disinfection of all contaminated supplies and equipment after a patient has been discharged to prepare them for reuse by another patient. The purpose of protective (reverse) isolation is to prevent a person with seriously impaired resistance from coming into contact who potentially pathogenic organisms.

Correct Q.27)

82% answered this correctly. 18% answered wrong

In which step of the nursing process would the nurse ask a patient if the medication she administered relieved his pain? A. Assessment B. Analysis C. Planning D. Evaluation (your answer) Explanation In the evaluation step of the nursing process, the nurse must decide whether the patient has achieved the expected outcome that was identified in the planning phase.

Correct Q.28)

84% answered this correctly. 16% answered wrong

Which of the following is a primary nursing intervention necessary for all patients with a Foley Catheter in place? A. Maintain the drainage tubing and collection bag level with the patient‟s bladder B. Irrigate the patient with 1% Neosporin solution three times a daily C. Clamp the catheter for 1 hour every 4 hours to maintain the bladder‟s elasticity D. Maintain the drainage tubing and collection bag below bladder level to facilitate drainage by gravity (your answer) Explanation Maintaing the drainage tubing and collection bag level with the patient‟s bladder could result in reflux of urine into the kidney. Irrigating the bladder with Neosporin and clamping the catheter for 1 hour every 4 hours must be prescribed by a physician.

Correct Q.29)

64% answered this correctly. 36% answered wrong

Parenteral penicillin can be administered as an: A. IM injection or an IV solution (your answer) B. IV or an intradermal injection C. Intradermal or subcutaneous injection D. IM or a subcutaneous injection Explanation Parenteral penicillin can be administered I.M. or added to a solution and given I.V. It cannot be administered subcutaneously or intradermally.

Correct Q.30)

32% answered this correctly. 68% answered wrong

The physician orders gr 10 of aspirin for a patient. The equivalent dose in milligrams is: A. 0.6 mg B. 10 mg C. 60 mg D. 600 mg (your answer) Explanation gr 10 x 60mg/gr 1 = 600 mg

Incorrect Q.31)

The mid-deltoid injection site is seldom used for I.M. injections because it:

61% answered this correctly. 39% answered wrong

A. Can accommodate only 1 ml or less of medication (correct answer) B. Bruises too easily (your answer) C. Can be used only when the patient is lying down D. Does not readily parenteral medication Explanation The mid-deltoid injection site can accommodate only 1 ml or less of medication because of its size and location (on the deltoid muscle of the arm, close to the brachial artery and radial nerve). Correct Q.32)

53% answered this correctly. 47% answered wrong

The appropriate needle size for insulin injection is: A. 18G, 1 ½” long B. 22G, 1” long C. 22G, 1 ½” long D. 25G, 5/8” long (your answer) Explanation A 25G, 5/8” needle is the recommended size for insulin injection because insulin is administered by the subcutaneous route. An 18G, 1 ½” needle is usually used for I.M. injections in children, typically in the vastus lateralis. A 22G, 1 ½” needle is usually used for adult I.M. injections, which are typically administered in the vastus lateralis or ventrogluteal site.

Incorrect Q.33)

87% answered this correctly. 13% answered wrong

Which of the following blood tests should be performed before a blood transfusion? A. Prothrombin and coagulation time B. Blood typing and cross-matching (correct answer) C. Bleeding and clotting time D. Complete blood count (CBC) and electrolyte levels. (your answer) Explanation Before a blood transfusion is performed, the blood of the donor and recipient must be checked for compatibility. This is done by blood typing (a test that determines a person‟s blood type) and cross-matching (a procedure that determines the compatibility of the donor‟s and recipient‟s blood after the blood types has been matched). If the blood specimens are incompatible, hemolysis and antigen-antibody reactions will occur.

Incorrect Q.34)

37% answered this correctly. 63% answered wrong

The primary purpose of a platelet count is to evaluate the: A. Potential for clot formation (correct answer) B. Potential for bleeding C. Presence of an antigen-antibody response (your answer) D. Presence of cardiac enzymes Explanation Platelets are disk-shaped cells that are essential for blood coagulation. A platelet count determines the number of thrombocytes in blood available for promoting hemostasis and assisting with blood coagulation after injury. It also is used to evaluate the patient‟s potential for bleeding; however, this is not its primary purpose. The normal count ranges from 150,000 to 350,000/mm3. A count of 100,000/mm3 or less indicates a potential for bleeding; count of less than 20,000/mm3 is associated with spontaneous bleeding.

Correct Q.35)

61% answered this correctly. 39% answered wrong

Which of the following white blood cell (WBC) counts clearly indicates leukocytosis? A. 4,500/mm³ B. 7,000/mm³ C. 10,000/mm³ D. 25,000/mm³ (your answer) Explanation Leukocytosis is any transient increase in the number of white blood cells (leukocytes) in the blood. Normal WBC counts range from 5,000 to 10,000/mm3. Thus, a count of 25,000/mm3 indicates leukocytosis.

Incorrect Q.36)

After routine patient contact, hand washing should last at least: A. 30 seconds (correct answer) B. 1 minute

47% answered this correctly. 53% answered wrong

C. 2 minute (your answer) D. 3 minutes Explanation Depending on the degree of exposure to pathogens, hand washing may last from 10 seconds to 4 minutes. After routine patient contact, hand washing for 30 seconds effectively minimizes the risk of pathogen transmission. Correct Q.37)

74% answered this correctly. 26% answered wrong

Which of the following patients is at greater risk for contracting an infection? A. A patient with leukopenia (your answer) B. A patient receiving broad-spectrum antibiotics C. A postoperative patient who has undergone orthopedic surgery D. A newly diagnosed diabetic patient Explanation Leukopenia is a decreased number of leukocytes (white blood cells), which are important in resisting infection. None of the other situations would put the patient at risk for contracting an infection; taking broadspectrum antibiotics might actually reduce the infection risk.

Correct Q.38)

32% answered this correctly. 68% answered wrong

Which of the following is a sign or symptom of a hemolytic reaction to blood transfusion? A. Hemoglobinuria (your answer) B. Chest pain C. Urticaria D. Distended neck veins Explanation Hemoglobinuria, the abnormal presence of hemoglobin in the urine, indicates a hemolytic reaction (incompatibility of the donor‟s and recipient‟s blood). In this reaction, antibodies in the recipient‟s plasma combine rapidly with donor RBC‟s; the cells are hemolyzed in either circulatory or reticuloendothelial system. Hemolysis occurs more rapidly in ABO incompatibilities than in Rh incompatibilities. Chest pain and urticaria may be symptoms of impending anaphylaxis. Distended neck veins are an indication of hypervolemia.

Correct Q.39)

73% answered this correctly. 27% answered wrong

All of the following measures are recommended to prevent pressure ulcers except: A. Massaging the reddened area with lotion (your answer) B. Using a water or air mattress C. Adhering to a schedule for positioning and turning D. Providing meticulous skin care Explanation Nurses and other health care professionals previously believed that massaging a reddened area with lotion would promote venous return and reduce edema to the area. However, research has shown that massage only increases the likelihood of cellular ischemia and necrosis to the area.

Correct Q.40)

64% answered this correctly. 36% answered wrong

A patient has returned to his room after femoral arteriography. All of the following are appropriate nursing interventions except: A. Assess femoral, popliteal, and pedal pulses every 15 minutes for 2 hours B. Check the pressure dressing for sanguineous drainage C. Assess a vital signs every 15 minutes for 2 hours D. Order a hemoglobin and hematocrit count 1 hour after the arteriography (your answer) Explanation A hemoglobin and hematocrit count would be ordered by the physician if bleeding were suspected. The other answers are appropriate nursing interventions for a patient who has undergone femoral arteriography.

Correct Q.41)

All of the following are common signs and symptoms of phlebitis except: A. Pain or discomfort at the IV insertion site B. Edema and warmth at the IV insertion site C. A red streak exiting the IV insertion site D. Frank bleeding at the insertion site (your answer) Explanation

68% answered this correctly. 32% answered wrong

Phlebitis, the inflammation of a vein, can be caused by chemical irritants (I.V. solutions or medications), mechanical irritants (the needle or catheter used during venipuncture or cannulation), or a localized allergic reaction to the needle or catheter. Signs and symptoms of phlebitis include pain or discomfort, edema and heat at the I.V. insertion site, and a red streak going up the arm or leg from the I.V. insertion site. Correct Q.42)

53% answered this correctly. 47% answered wrong

Which of the following procedures always requires surgical asepsis? A. Vaginal instillation of conjugated estrogen B. Urinary catheterization (your answer) C. Nasogastric tube insertion D. Colostomy irrigation Explanation The urinary system is normally free of microorganisms except at the urinary meatus. Any procedure that involves entering this system must use surgically aseptic measures to maintain a bacteria-free state.

Correct Q.43)

66% answered this correctly. 34% answered wrong

Clay colored stools indicate: A. Upper GI bleeding B. Impending constipation C. An effect of medication D. Bile obstruction (your answer) Explanation Bile colors the stool brown. Any inflammation or obstruction that impairs bile flow will affect the stool pigment, yielding light, clay-colored stool. Upper GI bleeding results in black or tarry stool. Constipation is characterized by small, hard masses. Many medications and foods will discolor stool – for example, drugs containing iron turn stool black.; beets turn stool red.

Incorrect Q.44)

39% answered this correctly. 61% answered wrong

Which of the following types of medications can be administered via gastrostomy tube? A. Any oral medications B. Capsules whole contents are dissolve in water C. Enteric-coated tablets that are thoroughly dissolved in water(your answer) D. Most tablets designed for oral use, except for extended-duration compounds (correct answer) Explanation Capsules, enteric-coated tablets, and most extended duration or sustained release products should not be dissolved for use in a gastrostomy tube. They are pharmaceutically manufactured in these forms for valid reasons, and altering them destroys their purpose. The nurse should seek an alternate physician‟s order when an ordered medication is inappropriate for delivery by tube.

Correct Q.45)

60% answered this correctly. 40% answered wrong

The best way of determining whether a patient has learned to instill ear medication properly is for the nurse to: A. Ask the patient if he/she has used ear drops before B. Have the patient repeat the nurse‟s instructions using her own words C. Demonstrate the procedure to the patient and encourage to ask questions D. Ask the patient to demonstrate the procedure (your answer) Explanation Return demonstration provides the most certain evidence for evaluating the effectiveness of patient teaching.

Incorrect Q.46)

63% answered this correctly. 37% answered wrong

When transferring a patient from a bed to a chair, the nurse should use which muscles to avoid back injury? A. Abdominal muscles B. Back muscles C. Leg muscles (correct answer) D. Upper arm muscles (your answer) Explanation The leg muscles are the strongest muscles in the body and should bear the greatest stress when lifting. Muscles of the abdomen, back, and upper arms may be easily injured.

Incorrect Q.47)

51% answered this correctly. 49% answered wrong

When removing a contaminated gown, the nurse should be careful that the first thing she touches is the:

A. Waist tie and neck tie at the back of the gown (correct answer) B. Waist tie in front of the gown C. Cuffs of the gown D. Inside of the gown (your answer) Explanation The back of the gown is considered clean, the front is contaminated. So, after removing gloves and washing hands, the nurse should untie the back of the gown; slowly move backward away from the gown, holding the inside of the gown and keeping the edges off the floor; turn and fold the gown inside out; discard it in a contaminated linen container; then wash her hands again. 71% answered this correctly. 29% answered wrong

Correct Q.48)

All of the following nursing interventions are correct when using the Ztrack method of drug injection except: A. Prepare the injection site with alcohol B. Use a needle that‟s a least 1” long C. Aspirate for blood before injection D. Rub the site vigorously after the injection to promote absorption(your answer) Explanation The Z-track method is an I.M. injection technique in which the patient‟s skin is pulled in such a way that the needle track is sealed off after the injection. This procedure seals medication deep into the muscle, thereby minimizing skin staining and irritation. Rubbing the injection site is contraindicated because it may cause the medication to extravasate into the skin.

Incorrect Q.49)

63% answered this correctly. 37% answered wrong

In a recumbent, immobilized patient, lung ventilation can become altered, leading to such respiratory complications as: A. Respiratory acidosis, ateclectasis, and hypostatic pneumonia(correct answer) B. Appneustic breathing, atypical pneumonia and respiratory alkalosis (your answer) C. Cheyne-Strokes respirations and spontaneous pneumothorax D. Kussmail‟s respirations and hypoventilation Explanation Because of restricted respiratory movement, a recumbent, immobilize patient is at particular risk for respiratory acidosis from poor gas exchange; atelectasis from reduced surfactant and accumulated mucus in the bronchioles, and hypostatic pneumonia from bacterial growth caused by stasis of mucus secretions.

Incorrect Q.50)

42% answered this correctly. 58% answered wrong

All of the following are good sources of vitamin A except: A. White potatoes (correct answer) B. Carrots C. Apricots D. Egg yolks (your answer) Explanation The main sources of vitamin A are yellow and green vegetables (such as carrots, sweet potatoes, squash, spinach, collard greens, broccoli, and cabbage) and yellow fruits (such as apricots, and cantaloupe). Animal sources include liver, kidneys, cream, butter, and egg yolks.

Correct Q.1)

35% answered this correctly. 65% answered wrong

Lydia is scheduled for elective splenectomy. Before the clients goes to surgery, the nurse in charge final assessment would be: A. signed consent B. vital signs (your answer) C. name band D. empty bladder Explanation An elective procedure is scheduled in advance so that all preparations can be completed ahead of time. The vital signs are the final check that must be completed before the client leaves the room so that continuity of care and assessment is provided for.

Correct

72% answered this correctly. 28% answered wrong

Q.2)

A male client with tuberculosis asks Nurse Brian how long the chemotherapy must be continued. Nurse Brian‟s accurate reply would be: A. 1 to 3 weeks B. 6 to 12 months (your answer) C. 3 to 5 months D. 3 years and more Explanation Tubercle bacillus is a drug resistant organism and takes a long time to be eradicated. Usually a combination of three drugs is used for minimum of 6 months and at least six months beyond culture conversion. 63% answered this correctly. 37% answered wrong

Correct Q.3)

What is the peak age range in acquiring acute lymphocytic leukemia (ALL)? A. 4 to 12 years. (your answer) B. 20 to 30 years C. 40 to 50 years D. 60 to 70 years Explanation The peak incidence of Acute Lymphocytic Leukemia (ALL) is 4 years of age. It is uncommon after 15 years of age. 54% answered this correctly. 46% answered wrong

Incorrect Q.4)

Which of the following stage the carcinogen is irreversible? A. Progression stage (correct answer) B. Initiation stage C. Regression stage (your answer) D. Promotion stage Explanation Progression stage is the change of tumor from the preneoplastic state or low degree of malignancy to a fast growing tumor that cannot be reversed. 53% answered this correctly. 47% answered wrong

Correct Q.5)

Nurse Maureen is aware that a client who has been diagnosed with chronic renal failure recognizes an adequate amount of high-biologic-value protein when the food the client selected from the menu was: A. Raw carrots B. Apple juice C. Whole wheat bread D. Cottage cheese (your answer) Explanation One cup of cottage cheese contains approximately 225 calories, 27 g of protein, 9 g of fat, 30 mg cholesterol, and 6 g of carbohydrate. Proteins of high biologic value (HBV) contain optimal levels of amino acids essential for life. 39% answered this correctly. 61% answered wrong

Correct Q.6)

A client is experiencing spinal shock. Nurse Myrna should expect the function of the bladder to be which of the following? A. Normal B. Atonic (your answer) C. Spastic D. Uncontrolled Explanation In spinal shock, the bladder becomes completely atonic and will continue to fill unless the client is catheterized. 82% answered this correctly. 18% answered wrong

Correct Q.7)

Among the following clients, which among them is high risk for potential hazards from the surgical experience? A. B. C. D.

67-year-old client (your answer) 49-year-old client 33-year-old client 15-year-old client

Explanation A 67 year old client is greater risk because the older adult client is more likely to have a less-effective immune system. Correct Q.8)

41% answered this correctly. 59% answered wrong

Halfway through the administration of blood, the female client complains of lumbar pain. After stopping the infusion Nurse Hazel should: A. Increase the flow of normal saline (your answer) B. Assess the pain further C. Notify the blood bank D. Obtain vital signs. Explanation The blood must be stopped at once, and then normal saline should be infused to keep the line patent and maintain blood volume.

Incorrect Q.9)

62% answered this correctly. 38% answered wrong

What is the priority nursing assessment in the first 24 hours after admission of the client with thrombotic CVA? A. Pupil size and papillary response (correct answer) B. cholesterol level (your answer) C. Echocardiogram D. Bowel sounds Explanation It is crucial to monitor the pupil size and papillary response to indicate changes around the cranial nerves.

Correct Q.10)

74% answered this correctly. 26% answered wrong

Which of the following signs and symptoms would Nurse Maureen include in teaching plan as an early manifestation of laryngeal cancer? A. Stomatitis B. Airway obstruction C. Hoarseness (your answer) D. Dysphagia Explanation Early warning signs of laryngeal cancer can vary depending on tumor location. Hoarseness lasting 2 weeks should be evaluated because it is one of the most common warning signs.

Correct Q.11)

53% answered this correctly. 47% answered wrong

A client has undergone laryngectomy. The immediate nursing priority would be: A. Keep trachea free of secretions (your answer) B. Monitor for signs of infection C. Provide emotional support D. Promote means of communication Explanation Patent airway is the most priority; therefore removal of secretions is necessary.

Correct Q.12)

81% answered this correctly. 19% answered wrong

Nurse hazel receives emergency laboratory results for a client with chest pain and immediately informs the physician. An increased myoglobin level suggests which of the following? A. Liver disease B. Myocardial damage (your answer) C. Hypertension D. Cancer Explanation Detection of myoglobin is a diagnostic tool to determine whether myocardial damage has occurred.

Correct Q.13)

47% answered this correctly. 53% answered wrong

The nurse is aware the early indicator of hypoxia in the unconscious client is: A. Cyanosis

B. Increased respirations C. Hypertension D. Restlessness (your answer) Explanation Restlessness is an early indicator of hypoxia. The nurse should suspect hypoxia in unconscious client who suddenly becomes restless. Correct Q.14)

47% answered this correctly. 53% answered wrong

Atropine sulfate (Atropine) is contraindicated in all but one of the following client?

A. A client with high blood B. A client with bowel obstruction C. A client with glaucoma (your answer) D. A client with U.T.I Explanation Atropine sulfate is contraindicated with glaucoma patients because it increases intraocular pressure. Correct Q.15)

38% answered this correctly. 62% answered wrong

A client has been diagnosed with Disseminated Intravascular Coagulation (DIC). Which of the following is contraindicated with the client? A. Administering Heparin B. Administering Coumadin (your answer) C. Treating the underlying cause D. Replacing depleted blood products Explanation Disseminated Intravascular Coagulation (DIC) has not been found to respond to oral anticoagulants such as Coumadin.

Correct Q.16)

28% answered this correctly. 72% answered wrong

Marco who was diagnosed with brain tumor was scheduled for craniotomy. In preventing the development of cerebral edema after surgery, the nurse should expect the use of:

A. Diuretics B. Antihypertensive C. Steroids (your answer) D. Anticonvulsants Explanation Glucocorticoids (steroids) are used for their anti-inflammatory action, which decreases the development of edema. Correct Q.17)

44% answered this correctly. 56% answered wrong

Kenneth who has diagnosed with uremic syndrome has the potential to develop complications. Which among the following complications should the nurse anticipates: A. Flapping hand tremors (your answer) B. An elevated hematocrit level C. Hypotension D. Hypokalemia Explanation Elevation of uremic waste products causes irritation of the nerves, resulting in flapping hand tremors.

Incorrect Q.18)

46% answered this correctly. 54% answered wrong

A 22 year old client suffered from his first tonic-clonic seizure. Upon awakening the client asks the nurse, “What caused me to have a seizure?Which of the following would the nurse include in the primary cause of tonic clonic seizures in adults more the 20 years? A. Electrolyte imbalance B. Head trauma (correct answer) C. Epilepsy (your answer) D. Congenital defect Explanation

Trauma is one of the primary cause of brain damage and seizure activity in adults. Other common causes of seizure activity in adults include neoplasms, withdrawal from drugs and alcohol, and vascular disease. 63% answered this correctly. 37% answered wrong

Correct Q.19)

The following are lipid abnormalities. Which of the following is a risk factor for the development of atherosclerosis and PVD? A. High levels of low density lipid (LDL) cholesterol (your answer) B. High levels of high density lipid (HDL) cholesterol C. Low concentration triglycerides D. Low levels of LDL cholesterol. Explanation An increased in LDL cholesterol concentration has been documented at risk factor for the development of atherosclerosis. LDL cholesterol is not broken down into the liver but is deposited into the wall of the blood vessels.

Incorrect Q.20)

33% answered this correctly. 67% answered wrong

Nurse Josie should instruct the client to eat which of the following foods to obtain the best supply of Vitamin B12? A. dairy products (correct answer) B. vegetables C. Grains D. Broccoli (your answer) Explanation Good source of vitamin B12 are dairy products and meats. 62% answered this correctly. 38% answered wrong

Correct Q.21)

Which of the following complications associated with tracheostomy tube? A. Increased cardiac output B. Acute respiratory distress syndrome (ARDS) C. Increased blood pressure D. Damage to laryngeal nerves (your answer) Explanation Tracheostomy tube has several potential complications including bleeding, infection and laryngeal nerve damage. 71% answered this correctly. 29% answered wrong

Correct

A client has been diagnosed with hypertension. The nurse priority nursing diagnosis would be:

Q.22)

A. Ineffective health maintenance (your answer) B. Impaired skin integrity C. Deficient fluid volume D. Pain Explanation Managing hypertension is the priority for the client with hypertension. Clients with hypertension frequently do not experience pain, deficient volume, or impaired skin integrity. It is the asymptomatic nature of hypertension that makes it so difficult to treat. Correct Q.23)

55% answered this correctly. 45% answered wrong

Nurse Katrina would recognize that the demonstration of crutch walking with tripod gait was understood when the client places weight on the: A. Palms of the hands and axillary regions B. Palms of the hand (your answer) C. Axillary regions D. Feet, which are set apart Explanation The palms should bear the client‟s weight to avoid damage to the nerves in the axilla.

Incorrect Q.24)

35% answered this correctly. 65% answered wrong

Nurse Katrina should anticipate that all of the following drugs may be used in the attempt to control the symptoms of Meniere's disease except:

A. Antiemetics B. Diuretics C. Antihistamines (your answer) D. Glucocorticoids (correct answer) Explanation Glucocorticoids play no significant role in disease treatment. 60% answered this correctly. 40% answered wrong

Correct Q.25)

A 65 year old female is experiencing flare up of pruritus. Which of the client‟s action could aggravate the cause of flare ups? A. Sleeping in cool and humidified environment B. Daily baths with fragrant soap (your answer) C. Using clothes made from 100% cotton D. Increasing fluid intake Explanation The use of fragrant soap is very drying to skin hence causing the pruritus. 57% answered this correctly. 43% answered wrong

Correct Q.26)

A male client‟s left tibia is fractures in an automobile accident, and a cast is applied. To assess for damage to major blood vessels from the fracture tibia, the nurse in charge should monitor the client for: A. Swelling of the left thigh B. Increased skin temperature of the foot C. Prolonged reperfusion of the toes after blanching (your answer) D. Increased blood pressure Explanation Damage to blood vessels may decrease the circulatory perfusion of the toes, this would indicate the lack of blood supply to the extremity.

Incorrect Q.27)

33% answered this correctly. 67% answered wrong

Mang Jose with rheumatoid arthritis states, “the only time I am without pain is when I lie in bed perfectly still”. During the convalescent stage, the nurse in charge with Mang Jose should encourage: A. Active joint flexion and extension (correct answer) B. Continued immobility until pain subsides C. Range of motion exercises twice daily (your answer) D. Flexion exercises three times daily Explanation Active exercises, alternating extension, flexion, abduction, and adduction, mobilize exudates in the joints relieves stiffness and pain. 24% answered this correctly. 76% answered wrong

Correct Q.28)

Nurse Jon assesses vital signs on a client undergone epidural anesthesia.Which of the following would the nurse assess next? A. Headache B. Bladder distension (your answer) C. Dizziness D. Ability to move legs Explanation The last area to return sensation is in the perineal area, and the nurse in charge should monitor the client for distended bladder.

Incorrect Q.29)

55% answered this correctly. 45% answered wrong

A male client has undergone spinal surgery, the nurse should: A. Observe the client‟s bowel movement and voiding patterns(your answer) B. Log-roll the client to prone position C. Assess the client‟s feet for sensation and circulation (correct answer) D. Encourage client to drink plenty of fluids Explanation Alteration in sensation and circulation indicates damage to the spinal cord, if these occurs notify physician immediately.

Incorrect Q.30)

52% answered this correctly. 48% answered wrong

A client has undergone with penile implant. After 24 hrs of surgery, the client‟s scrotum was edematous and painful. The nurse should: A. Assist the client with sitz bath B. Apply war soaks in the scrotum C. Elevate the scrotum using a soft support (correct answer) D. Prepare for a possible incision and drainage. (your answer) Explanation Elevation increases lymphatic drainage, reducing edema and pain.

Correct Q.31)

87% answered this correctly. 13% answered wrong

Nurse Maureen knows that the positive diagnosis for HIV infection is made based on which of the following: A. A history of high risk sexual behaviors. B. Positive ELISA and western blot tests (your answer) C. Identification of an associated opportunistic infection D. Evidence of extreme weight loss and high fever Explanation These tests confirm the presence of HIV antibodies that occur in response to the presence of the human immunodeficiency virus (HIV).

Correct Q.32)

47% answered this correctly. 53% answered wrong

Nurse Faith should recognize that fluid shift in an client with burn injury results from increase in the: A. Total volume of circulating whole blood B. Total volume of intravascular plasma C. Permeability of capillary walls (your answer) D. Permeability of kidney tubules Explanation In burn, the capillaries and small vessels dilate, and cell damage cause the release of a histamine-like substance. The substance causes the capillary walls to become more permeable and significant quantities of fluid are lost.

Correct Q.33)

72% answered this correctly. 28% answered wrong

Karen has been diagnosed with aplastic anemia. The nurse monitors for changes in which of the following physiologic functions? A. Bowel function B. Peripheral sensation C. Bleeding tendencies (your answer) D. Intake and out put Explanation Aplastic anemia decreases the bone marrow production of RBC‟s, white blood cells, and platelets. The client is at risk for bruising and bleeding tendencies.

Correct Q.34)

34% answered this correctly. 66% answered wrong

After a long leg cast is removed, the male client should: A. Cleanse the leg by scrubbing with a brisk motion B. Put leg through full range of motion twice daily C. Report any discomfort or stiffness to the physician D. Elevate the leg when sitting for long periods of time. (your answer) Explanation Elevation will help control the edema that usually occurs.

Correct Q.35)

48% answered this correctly. 52% answered wrong

Which of the following represents a significant risk immediately after surgery for repair of aortic aneurysm?

A. Potential wound infection B. Potential ineffective coping

C. Potential electrolyte balance D. Potential alteration in renal perfusion (your answer) Explanation There is a potential alteration in renal perfusion manifested by decreased urine output. The altered renal perfusion may be related to renal artery embolism, prolonged hypotension, or prolonged aortic cross-clamping during the surgery. Incorrect

64% answered this correctly. 36% answered wrong

Q.36) Nurse Hazel teaches the client with angina about common expected side effects of nitroglycerin including: high blood pressure (your answer) A. stomach cramps B. headache (correct answer) C. shortness of breath D. Explanation Because of its widespread vasodilating effects, nitroglycerin often produces side effects such as headache, hypotension and dizziness. 37% answered this correctly. 63% answered wrong

Correct Q.37)

Nurse Maureen would expect the a client with mitral stenosis would demonstrate symptoms associated with congestion in the: A. Right atrium B. Superior vena cava C. Aorta D. Pulmonary (your answer) Explanation When mitral stenosis is present, the left atrium has difficulty emptying its contents into the left ventricle because there is no valve to prevent back ward flow into the pulmonary vein, the pulmonary circulation is under pressure.

Incorrect

45% answered this correctly. 55% answered wrong

Nurse Anna is aware that early adaptation of client with renal carcinoma is:

Q.38)

A. Nausea and vomiting B. flank pain C. weight gain (your answer) D. intermittent hematuria (correct answer) Explanation Intermittent pain is the classic sign of renal carcinoma. It is primarily due to capillary erosion by the cancerous growth. 60% answered this correctly. 40% answered wrong

Correct Q.39)

An 83-year-old woman has several ecchymotic areas on her right arm. The bruises are probably caused by: A. increased capillary fragility and permeability (your answer) B. increased blood supply to the skin C. self inflicted injury D. elder abuse Explanation Aging process involves increased capillary fragility and permeability. Older adults have a decreased amount of subcutaneous fat and cause an increased incidence of bruise like lesions caused by collection of extravascular blood in loosely structured dermis.

Incorrect

65% answered this correctly. 35% answered wrong

A female client is receiving IV Mannitol. An assessment specific to safe administration of the said drug is:

Q.40)

A. Vital signs q4h B. Weighing daily (your answer) – effectiveness? C. Urine output hourly (correct answer) D. Level of consciousness q4h Explanation The osmotic diuretic mannitol is contraindicated in the presence of inadequate renal function or heart failure because it increases the intravascular volume that must be filtered and excreted by the kidney. Correct

70% answered this correctly. 30% answered wrong

Q.41)

Which of the following findings is the best indication that fluid replacement for the client with hypovolemic shock is adequate? A. Urine output greater than 30ml/hr (your answer) B. Respiratory rate of 21 breaths/minute C. Diastolic blood pressure greater than 90 mmhg D. Systolic blood pressure greater than 110 mmhg Explanation Urine output provides the most sensitive indication of the client‟s response to therapy for hypovolemic shock. Urine output should be consistently greater than 30 to 35 mL/hr. 63% answered this correctly. 37% answered wrong

Correct Q.42)

Nurse Judith obtains a specimen of clear nasal drainage from a client with a head injury. Which of the following tests differentiates mucus from cerebrospinal fluid (CSF)? A. Protein B. Specific gravity C. Glucose (your answer) D. Microorganism Explanation The constituents of CSF are similar to those of blood plasma. An examination for glucose content is done to determine whether a body fluid is a mucus or a CSF. A CSF normally contains glucose. 32% answered this correctly. 68% answered wrong

Correct

A client is admitted to the hospital with benign prostatic hyperplasia, the nurse most relevant assessment would be:

Q.43)

A. Flank pain radiating in the groin B. Distention of the lower abdomen (your answer) C. Perineal edema D. Urethral discharge Explanation This indicates that the bladder is distended with urine, therefore palpable. 66% answered this correctly. 34% answered wrong

Correct

Among the following components thorough pain assessment, which is the most significant?

Q.44)

A. Effect B. Cause C. Causing factors D. Intensity (your answer) Explanation Intensity is the major indicative of severity of pain and it is important for the evaluation of the treatment. Correct Q.45)

47% answered this correctly. 53% answered wrong

Karina a client with myasthenia gravis is to receive immunosuppressive therapy. The nurse understands that this therapy is effective because it: A. Promotes the removal of antibodies that impair the transmission of impulses B. Stimulates the production of acetylcholine at the neuromuscular junction. C. Decreases the production of autoantibodies that attack the acetylcholine receptors. (your answer) D. Inhibits the breakdown of acetylcholine at the neuromuscular junction. Explanation Steroids decrease the body‟s immune response thus decreasing the production of antibodies that attack the acetylcholine receptors at the neuromuscular junction

Correct Q.46)

41% answered this correctly. 59% answered wrong

Marie with acute lymphocytic leukemia suffers from nausea and headache. These clinical manifestations may indicate all of the following except A. effects of radiation B. chemotherapy side effects C. meningeal irritation

D. gastric distension (your answer) Explanation Acute Lymphocytic Leukemia (ALL) does not cause gastric distention. It does invade the central nervous system, and clients experience headaches and vomiting from meningeal irritation. 65% answered this correctly. 35% answered wrong

Correct

Nurse Linda is preparing a client with multiple sclerosis for discharge from the hospital to home. Which of the following instruction is most appropriate?

Q.47)

A. “Practice using the mechanical aids that you will need when future disabilities arise”. B. “Follow good health habits to change the course of the disease”. C. “Keep active, use stress reduction strategies, and avoid fatigue.(your answer) D. “You will need to accept the necessity for a quiet and inactive lifestyle”. Explanation The nurse most positive approach is to encourage the client with multiple sclerosis to stay active, use stress reduction techniques and avoid fatigue because it is important to support the immune system while remaining active. 51% answered this correctly. 49% answered wrong

Correct

Marina with acute renal failure moves into the diuretic phase after one week of therapy. During this phase the client must be assessed for signs of developing:

Q.48)

A. Hypovolemia (your answer) B. renal failure C. metabolic acidosis D. hyperkalemia Explanation In the diuretic phase fluid retained during the oliguric phase is excreted and may reach 3 to 5 liters daily, hypovolemia may occur and fluids should be replaced. 49% answered this correctly. 51% answered wrong

Correct

While performing a physical assessment of a male client with gout of the great toe, NurseVivian should assess for additional tophi (urate deposits) on the:

Q.49)

A. Buttocks B. Ears (your answer) C. Face D. Abdomen Explanation Uric acid has a low solubility, it tends to precipitate and form deposits at various sites where blood flow is least active, including cartilaginous tissue such as the ears. 63% answered this correctly. 37% answered wrong

Incorrect

Patricia a 20 year old college student with diabetes mellitus requests additional information about the advantages of using a pen like insulin delivery devices. The nurse explains that the advantages of these devices over syringes includes:

Q.50)

A. Accurate dose delivery (correct answer) B. Shorter injection time C. Lower cost with reusable insulin cartridges D. Use of smaller gauge needle. (your answer) Explanation These devices are more accurate because they are easily to used and have improved adherence in insulin regimens by young people because the medication can be administered discreetly.

56% answered this correctly. 44% answered wrong

Correct Q.1)

The nurse hears a mother telling a friend on the telephone about umbilical cord care. Which of the following statements by the mother indicates effective teaching? A. B. C. D.

“Daily soap and water cleansing is best” „Alcohol helps it dry and kills germs” (your answer) “An antibiotic ointment applied daily prevents infection” “He can have a tub bath each day”

Explanation Application of 70% isopropyl alcohol to the cord minimizes microorganisms (germicidal) and promotes drying. The cord should be kept dry until it falls off and the stump has healed. Antibiotic ointment should only be used to treat an infection, not as a prophylaxis. Infants should not be submerged in a tub of water until the cord falls off and the stump has completely healed. Incorrect Q.2)

46% answered this correctly. 54% answered wrong

When developing a plan of care for a client newly diagnosed with gestational diabetes, which of the following instructions would be the priority? A. Dietary intake (correct answer) B. Medication C. Exercise D. Glucose monitoring (your answer) Explanation Although all of the choices are important in the management of diabetes, diet therapy is the mainstay of the treatment plan and should always be the priority. Women diagnosed with gestational diabetes generally need only diet therapy without medication to control their blood sugar levels. Exercise, is important for all pregnant women and especially for diabetic women, because it burns up glucose, thus decreasing blood sugar. However, dietary intake, not exercise, is the priority. All pregnant women with diabetes should have periodic monitoring of serum glucose. However, those with gestational diabetes generally do not need daily glucose monitoring. The standard of care recommends a fasting and 2- hour postprandial blood sugar level every 2 weeks.

Correct Q.3)

69% answered this correctly. 31% answered wrong

When teaching a group of adolescents about male hormone production, which of the following would the nurse include as being produced by the Leydig cells? A. Follicle-stimulating hormone B. Testosterone (your answer) C. Leuteinizing hormone D. Gonadotropin releasing hormone Explanation Testosterone is produced by the Leyding cells in the seminiferous tubules. Follicle-stimulating hormone and leuteinzing hormone are released by the anterior pituitary gland. The hypothalamus is responsible for releasing gonadotropin-releasing hormone.

Correct Q.4)

39% answered this correctly. 61% answered wrong

To differentiate as a female, the hormonal stimulation of the embryo that must occur involves which of the following? A. Increase in maternal estrogen secretion B. Decrease in maternal androgen secretion C. Secretion of androgen by the fetal gonad D. Secretion of estrogen by the fetal gonad (your answer) Explanation The fetal gonad must secrete estrogen for the embryo to differentiate as a female. An increase in maternal estrogen secretion does not effect differentiation of the embryo, and maternal estrogen secretion occurs in every pregnancy. Maternal androgen secretion remains the same as before pregnancy and does not effect differentiation. Secretion of androgen by the fetal gonad would produce a male fetus.

Correct Q.5)

61% answered this correctly. 39% answered wrong

Before assessing the postpartum client‟s uterus for firmness and position in relation to the umbilicus and midline, which of the following should the nurse do first? A. Assess the vital signs B. Administer analgesia C. Ambulate her in the hall D. Assist her to urinate (your answer) Explanation Before uterine assessment is performed, it is essential that the woman empty her bladder. A full bladder will interfere with the accuracy of the assessment by elevating the uterus and displacing to the side of the midline. Vital sign assessment is not necessary unless an abnormality in uterine assessment is identified. Uterine assessment should not cause acute pain that requires administration of analgesia. Ambulating the client is an essential component of postpartum care, but is not necessary prior to assessment of the uterus.

Correct Q.6)

56% answered this correctly. 44% answered wrong

A client has a midpelvic contracture from a previous pelvic injury due to a motor vehicle accident as a teenager. The nurse is aware that this could prevent a fetus from passing through or around which structure during childbirth?

A. Symphysis pubis B. Sacral promontory C. Ischial spines (your answer) D. Pubic arch Explanation The ischial spines are located in the mid-pelvic region and could be narrowed due to the previous pelvic injury. The symphysis pubis, sacral promontory, and pubic arch are not part of the mid-pelvis. Incorrect Q.7)

62% answered this correctly. 38% answered wrong

When describing dizygotic twins to a couple, on which of the following would the nurse base the explanation? A. Two ova fertilized by separate sperm (correct answer) B. Sharing of a common placenta C. Each ova with the same genotype (your answer) D. Sharing of a common chorion Explanation Dizygotic (fraternal) twins involve two ova fertilized by separate sperm. Monozygotic (identical) twins involve a common placenta, same genotype, and common chorion.

Correct Q.8)

85% answered this correctly. 15% answered wrong

When preparing to administer the vitamin K injection to a neonate, the nurse would select which of the following sites as appropriate for the injection? A. Deltoid muscle B. Anterior femoris muscle C. Vastus lateralis muscle (your answer) D. Gluteus maximus muscle Explanation The middle third of the vastus lateralis is the preferred injection site for vitamin K administration because it is free of blood vessels and nerves and is large enough to absorb the medication. The deltoid muscle of a newborn is not large enough for a newborn IM injection. Injections into this muscle in a small child might cause damage to the radial nerve. The anterior femoris muscle is the next safest muscle to use in a newborn but is not the safest. Because of the proximity of the sciatic nerve, the gluteus maximus muscle should not be until the child has been walking 2 years.

Incorrect Q.9)

58% answered this correctly. 42% answered wrong

During a pelvic exam the nurse notes a purple-blue tinge of the cervix. The nurse documents this as which of the following? A. Braxton-Hicks sign B. Chadwick‟s sign (correct answer) C. Goodell‟s sign (your answer) D. McDonald‟s sign Explanation Chadwick‟s sign refers to the purple-blue tinge of the cervix. Braxton Hicks contractions are painless contractions beginning around the 4th month. Goodell‟s sign indicates softening of the cervix. Flexibility of the uterus against the cervix is known as McDonald‟s sign.

Incorrect Q.10)

24% answered this correctly. 76% answered wrong

In the late 1950s, consumers and health care professionals began challenging the routine use of analgesics and anesthetics during childbirth. Which of the following was an outgrowth of this concept? A. Labor, delivery, recovery, postpartum (LDRP) (your answer) B. Nurse-midwifery C. Clinical nurse specialist D. Prepared childbirth (correct answer) Explanation Prepared childbirth was the direct result of the 1950‟s challenging of the routine use of analgesic and anesthetics during childbirth. The LDRP was a much later concept and was not a direct result of the challenging of routine use of analgesics and anesthetics during childbirth. Roles for nurse midwives and clinical nurse specialists did not develop from this challenge.

Incorrect Q.11)

48% answered this correctly. 52% answered wrong

A client at 24 weeks gestation has gained 6 pounds in 4 weeks. Which of the following would be the priority when assessing the client?

A. Glucosuria (your answer) B. Depression C. Hand/face edema (correct answer) D. Dietary intake Explanation After 20 weeks‟ gestation, when there is a rapid weight gain, preeclampsia should be suspected, which may be caused by fluid retention manifested by edema, especially of the hands and face. The three classic signs of preeclampsia are hypertension, edema, and proteinuria. Although urine is checked for glucose at each clinic visit, this is not the priority. Depression may cause either anorexia or excessive food intake, leading to excessive weight gain or loss. This is not, however, the priority consideration at this time. Weight gain thought to be caused by excessive food intake would require a 24-hour diet recall. However, excessive intake would not be the primary consideration for this client at this time. Correct Q.12)

67% answered this correctly. 33% answered wrong

Which of the following would be the nurse‟s most appropriate response to a client who asks why she must have a cesarean delivery if she has a complete placenta previa? A. “You will have to ask your physician when he returns.” B. “You need a cesarean to prevent hemorrhage.” C. “The placenta is covering most of your cervix.” D. “The placenta is covering the opening of the uterus and blocking your baby.” (your answer) Explanation A complete placenta previa occurs when the placenta covers the opening of the uterus, thus blocking the passageway for the baby. This response explains what a complete previa is and the reason the baby cannot come out except by cesarean delivery. Telling the client to ask the physician is a poor response and would increase the patient‟s anxiety. Although a cesarean would help to prevent hemorrhage, the statement does not explain why the hemorrhage could occur. With a complete previa, the placenta is covering all the cervix, not just most of it.

Correct Q.13)

87% answered this correctly. 13% answered wrong

When teaching a client about contraception. Which of the following would the nurse include as the most effective method for preventing sexually transmitted infections? A. Spermicides B. Diaphragm C. Condoms (your answer) D. Vasectomy Explanation Condoms, when used correctly and consistently, are the most effective contraceptive method or barrier against bacterial and viral sexually transmitted infections. Although spermicides kill sperm, they do not provide reliable protection against the spread of sexually transmitted infections, especially intracellular organisms such as HIV. Insertion and removal of the diaphragm along with the use of the spermicides may cause vaginal irritations, which could place the client at risk for infection transmission. Male sterilization eliminates spermatozoa from the ejaculate, but it does not eliminate bacterial and/or viral microorganisms that can cause sexually transmitted infections.

Correct Q.14)

57% answered this correctly. 43% answered wrong

When performing a pelvic examination, the nurse observes a red swollen area on the right side of the vaginal orifice. The nurse would document this as enlargement of which of the following? A. Clitoris B. Parotid gland C. Skene‟s gland D. Bartholin‟s gland (your answer) Explanation Bartholin‟s glands are the glands on either side of the vaginal orifice. The clitoris is female erectile tissue found in the perineal area above the urethra. The parotid glands are open into the mouth. Skene‟s glands open into the posterior wall of the female urinary meatus.

Correct Q.15)

32% answered this correctly. 68% answered wrong

For which of the following clients would the nurse expect that an intrauterine device would not be recommended? . A. Woman over age 35 B. Nulliparous woman C. Promiscuous young adult (your answer)

D. Postpartum client Explanation An IUD may increase the risk of pelvic inflammatory disease, especially in women with more than one sexual partner, because of the increased risk of sexually transmitted infections. An UID should not be used if the woman has an active or chronic pelvic infection, postpartum infection, endometrial hyperplasia or carcinoma, or uterine abnormalities. Age is not a factor in determining the risks associated with IUD use. Most IUD users are over the age of 30. Although there is a slightly higher risk for infertility in women who have never been pregnant, the IUD is an acceptable option as long as the risk-benefit ratio is discussed. IUDs may be inserted immediately after delivery, but this is not recommended because of the increased risk and rate of expulsion at this time. Correct Q.16)

66% answered this correctly. 34% answered wrong

After 4 hours of active labor, the nurse notes that the contractions of a primigravida client are not strong enough to dilate the cervix. Which of the following would the nurse anticipate doing? A. Obtaining an order to begin IV oxytocin infusion (your answer) B. Administering a light sedative to allow the patient to rest for several hour C. Preparing for a cesarean section for failure to progress D. Increasing the encouragement to the patient when pushing begins Explanation The client‟s labor is hypotonic. The nurse should call the physical and obtain an order for an infusion of oxytocin, which will assist the uterus to contact more forcefully in an attempt to dilate the cervix. Administering light sedative would be done for hypertonic uterine contractions. Preparing for cesarean section is unnecessary at this time. Oxytocin would increase the uterine contractions and hopefully progress labor before a cesarean would be necessary. It is too early to anticipate client pushing with contractions.

Correct Q.17)

42% answered this correctly. 58% answered wrong

A client 12 weeks‟ pregnant come to the emergency department with abdominal cramping and moderate vaginal bleeding. Speculum examination reveals 2 to 3 cms cervical dilation. The nurse would document these findings as which of the following? A. Threatened abortion B. Imminent abortion (your answer) C. Complete abortion D. Missed abortion Explanation Cramping and vaginal bleeding coupled with cervical dilation signifies that termination of the pregnancy is inevitable and cannot be prevented. Thus, the nurse would document an imminent abortion. In a threatened abortion, cramping and vaginal bleeding are present, but there is no cervical dilation. The symptoms may subside or progress to abortion. In a complete abortion all the products of conception are expelled. A missed abortion is early fetal intrauterine death without expulsion of the products of conception.

Correct Q.18)

76% answered this correctly. 24% answered wrong

A multigravida at 38 weeks‟ gestation is admitted with painless, bright red bleeding and mild contractions every 7 to 10 minutes. Which of the following assessments should be avoided? A. Maternal vital sign B. Fetal heart rate C. Contraction monitoring D. Cervical dilation (your answer) Explanation The signs indicate placenta previa and vaginal exam to determine cervical dilation would not be done because it could cause hemorrhage. Assessing maternal vital signs can help determine maternal physiologic status. Fetal heart rate is important to assess fetal well-being and should be done. Monitoring the contractions will help evaluate the progress of labor.

Correct Q.19)

40% answered this correctly. 60% answered wrong

A newborn who has an asymmetrical Moro reflex response should be further assessed for which of the following? A. Talipes equinovarus B. Fractured clavicle (your answer) C. Congenital hypothyroidism D. Increased intracranial pressure Explanation A fractured clavicle would prevent the normal Moro response of symmetrical sequential extension and abduction of the arms followed by flexion and adduction. In talipes equinovarus (clubfoot) the foot is turned medially, and in plantar flexion, with the heel elevated. The feet are not involved with the Moro reflex. Hypothyroiddism has no effect on the primitive reflexes. Absence

of the Moror reflex is the most significant single indicator of central nervous system status, but it is not a sign of increased intracranial pressure. Correct Q.20)

64% answered this correctly. 36% answered wrong

A client with severe preeclampsia is admitted with of BP 160/110, proteinuria, and severe pitting edema. Which of the following would be most important to include in the client‟s plan of care? A. Daily weights B. Seizure precautions (your answer) C. Right lateral positioning D. Stress reduction Explanation Women hospitalized with severe preeclampsia need decreased CNS stimulation to prevent a seizure. Seizure precautions provide environmental safety should a seizure occur. Because of edema, daily weight is important but not the priority. Preclampsia causes vasospasm and therefore can reduce utero-placental perfusion. The client should be placed on her left side to maximize blood flow, reduce blood pressure, and promote diuresis. Interventions to reduce stress and anxiety are very important to facilitate coping and a sense of control, but seizure precautions are the priority.

Incorrect Q.21)

50% answered this correctly. 50% answered wrong

Which of the following refers to the single cell that reproduces itself after conception? A. Chromosome B. Blastocyst (your answer) C. Zygote (correct answer) D. Trophoblast Explanation The zygote is the single cell that reproduces itself after conception. The chromosome is the material that makes up the cell and is gained from each parent. Blastocyst and trophoblast are later terms for the embryo after zygote.

Correct Q.22)

65% answered this correctly. 35% answered wrong

Which of the following is the priority focus of nursing practice with the current early postpartum discharge? A. Promoting comfort and restoration of health B. Exploring the emotional status of the family C. Facilitating safe and effective self-and newborn care (your answer) D. Teaching about the importance of family planning Explanation Because of early postpartum discharge and limited time for teaching, the nurse‟s priority is to facilitate the safe and effective care of the client and newborn. Although promoting comfort and restoration of health, exploring the family‟s emotional status, and teaching about family planning are important in postpartum/newborn nursing care, they are not the priority focus in the limited time presented by early post-partum discharge.

Correct Q.23)

62% answered this correctly. 38% answered wrong

When taking an obstetrical history on a pregnant client who states, “I had a son born at 38 weeks gestation, a daughter born at 30 weeks gestation and I lost a baby at about 8 weeks,” the nurse should record her obstetrical history as which of the following? . A. G2 T2 P0 A0 L2 B. G3 T1 P1 A0 L2 C. G3 T2 P0 A0 L2 D. G4 T1 P1 A1 L2 (your answer) Explanation The client has been pregnant four times, including current pregnancy (G). Birth at 38 weeks‟ gestation is considered full term (T), while birth form 20 weeks to 38 weeks is considered preterm (P). A spontaneous abortion occurred at 8 weeks (A). She has two living children (L).

Correct Q.24)

40% answered this correctly. 60% answered wrong

With a fetus in the left-anterior breech presentation, the nurse would expect the fetal heart rate would be most audible in which of the following areas? A. Above the maternal umbilicus and to the right of midline B. In the lower-left maternal abdominal quadrant C. In the lower-right maternal abdominal quadrant

D. Above the maternal umbilicus and to the left of midline (your answer) Explanation With this presentation, the fetal upper torso and back face the left upper maternal abdominal wall. The fetal heart rate would be most audible above the maternal umbilicus and to the left of the middle. The other positions would be incorrect. Incorrect Q.25)

81% answered this correctly. 19% answered wrong

A patient is in labor and has just been told she has a breech presentation. The nurse should be particularly alert for which of the following? A. Quickening B. Ophthalmia neonatorum C. Pica (your answer) D. Prolapsed umbilical cord (correct answer) Explanation In a breech position, because of the space between the presenting part and the cervix, prolapse of the umbilical cord is common. Quickening is the woman‟s first perception of fetal movement. Ophthalmia neonatorum usually results from maternal gonorrhea and is conjunctivitis. Pica refers to the oral intake of nonfood substances.

Incorrect Q.26)

30% answered this correctly. 70% answered wrong

A client at 8 weeks‟ gestation calls complaining of slight nausea in the morning hours. Which of the following client interventions should the nurse question? A. Taking 1 teaspoon of bicarbonate of soda in an 8-ounce glass of water (correct answer) B. Eating a few low-sodium crackers before getting out of bed(your answer) C. Avoiding the intake of liquids in the morning hours D. Eating six small meals a day instead of thee large meals Explanation Using bicarbonate would increase the amount of sodium ingested, which can cause complications. Eating low-sodium crackers would be appropriate. Since liquids can increase nausea avoiding them in the morning hours when nausea is usually the strongest is appropriate. Eating six small meals a day would keep the stomach full, which often decrease nausea.

Correct Q.27)

64% answered this correctly. 36% answered wrong

When preparing to listen to the fetal heart rate at 12 weeks‟ gestation, the nurse would use which of the following? A. Stethoscope placed midline at the umbilicus B. Doppler placed midline at the suprapubic region (your answer) C. Fetoscope placed midway between the umbilicus and the xiphoid process D. External electronic fetal monitor placed at the umbilicus Explanation At 12 weeks gestation, the uterus rises out of the pelvis and is palpable above the symphysis pubis. The Doppler intensifies the sound of the fetal pulse rate so it is audible. The uterus has merely risen out of the pelvis into the abdominal cavity and is not at the level of the umbilicus. The fetal heart rate at this age is not audible with a stethoscope. The uterus at 12 weeks is just above the symphysis pubis in the abdominal cavity, not midway between the umbilicus and the xiphoid process. At 12 weeks the FHR would be difficult to auscultate with a fetoscope. Although the external electronic fetal monitor would project the FHR, the uterus has not risen to the umbilicus at 12 weeks.

Correct Q.28)

70% answered this correctly. 30% answered wrong

The nurse assesses the postpartum vaginal discharge (lochia) on four clients. Which of the following assessments would warrant notification of the physician? A. A dark red discharge on a 2-day postpartum client B. A pink to brownish discharge on a client who is 5 days postpartum C. Almost colorless to creamy discharge on a client 2 weeks after delivery D. A bright red discharge 5 days after delivery (your answer) Explanation Any bright red vaginal discharge would be considered abnormal, but especially 5 days after delivery, when the lochia is typically pink to brownish. Lochia rubra, a dark red discharge, is present for 2 to 3 days after delivery. Bright red vaginal bleeding at this time suggests late postpartum hemorrhage, which occurs after the first 24 hours following delivery and is generally caused by retained placental fragments or bleeding disorders. Lochia rubra is the normal dark red discharge occurring in the first 2 to 3 days after delivery, containing epithelial cells, erythrocyes, leukocytes and decidua. Lochia serosa is a pink to brownish serosanguineous discharge occurring from 3 to 10 days after delivery that contains decidua, erythrocytes, leukocytes, cervical mucus, and microorganisms. Lochia alba is an almost colorless to yellowish discharge occurring from 10 days to 3 weeks after delivery and containing leukocytes, decidua, epithelial cells, fat, cervical mucus, cholesterol crystals, and bacteria.

Correct Q.29)

57% answered this correctly. 43% answered wrong

Which of the following would be the priority nursing diagnosis for a client with an ectopic pregnancy? A. Risk for infection B. Pain (your answer) C. Knowledge Deficit D. Anticipatory Grieving Explanation For the client with an ectopic pregnancy, lower abdominal pain, usually unilateral, is the primary symptom. Thus, pain is the priority. Although the potential for infection is always present, the risk is low in ectopic pregnancy because pathogenic microorganisms have not been introduced from external sources. The client may have a limited knowledge of the pathology and treatment of the condition and will most likely experience grieving, but this is not the priority at this time.

Correct Q.30)

65% answered this correctly. 35% answered wrong

Which of the following actions would be least effective in maintaining a neutral thermal environment for the newborn? A. Placing infant under radiant warmer after bathing B. Covering the scale with a warmed blanket prior to weighing C. Placing crib close to nursery window for family viewing (your answer) D. Covering the infant‟s head with a knit stockinette Explanation Heat loss by radiation occurs when the infant‟s crib is placed too near cold walls or windows. Thus placing the newborn‟s crib close to the viewing window would be least effective. Body heat is lost through evaporation during bathing. Placing the infant under the radiant warmer after bathing will assist the infant to be rewarmed. Covering the scale with a warmed blanket prior to weighing prevents heat loss through conduction. A knit cap prevents heat loss from the head a large head, a large body surface area of the newborn‟s body.

Correct Q.31)

60% answered this correctly. 40% answered wrong

A postpartum client has a temperature of 101.4ºF, with a uterus that is tender when palpated, remains unusually large, and not descending as normally expected. Which of the following should the nurse assess next? A. Lochia (your answer) B. Breasts C. Incision D. Urine Explanation The data suggests an infection of the endometrial lining of the uterus. The lochia may be decreased or copious, dark brown in appearance, and foul smelling, providing further evidence of a possible infection. All the client‟s data indicate a uterine problem, not a breast problem. Typically, transient fever, usually 101ºF, may be present with breast engorgement. Symptoms of mastitis include influenza-like manifestations. Localized infection of an episiotomy or C-section incision rarely causes systemic symptoms, and uterine involution would not be affected. The client data do not include dysuria, frequency, or urgency, symptoms of urinary tract infections, which would necessitate assessing the client‟s urine.

Incorrect Q.32)

48% answered this correctly. 52% answered wrong

The mother asks the nurse. “What‟s wrong with my son‟s breasts? Why are they so enlarged?” Whish of the following would be the best response by the nurse? A. “The breast tissue is inflamed from the trauma experienced with birth” B. “A decrease in material hormones present before birth causes enlargement,” (correct answer) C. “You should discuss this with your doctor. It could be a malignancy” D. “The tissue has hypertrophied while the baby was in the uterus”(your answer) Explanation The presence of excessive estrogen and progesterone in the maternal fetal blood followed by prompt withdrawal at birth precipitates breast engorgement, which will spontaneously resolve in 4 to 5 days after birth. The trauma of the birth process does not cause inflammation of the newborn‟s breast tissue. Newborns do not have breast malignancy. This reply by the nurse would cause the mother to have undue anxiety. Breast tissue does not hypertrophy in the fetus or newborns.

Incorrect Q.33)

28% answered this correctly. 72% answered wrong

A newborn weighing 3000 grams and feeding every 4 hours needs 120 calories/kg of body weight every 24 hours for proper growth and development. How many ounces of 20 cal/oz formula should this newborn receive at each feeding to meet nutritional needs? A. 2 ounces

B. 3 ounces (correct answer) C. 4 ounces D. 6 ounces (your answer) Explanation To determine the amount of formula needed, do the following mathematical calculation. 3 kg x 120 cal/kg per day = 360 calories/day feeding q 4 hours = 6 feedings per day = 60 calories per feeding: 60 calories per feeding; 60 calories per feeding with formula 20 cal/oz = 3 ounces per feeding. Based on the calculation. 2, 4 or 6 ounces are incorrect. Correct Q.34)

87% answered this correctly. 13% answered wrong

A client in her third trimester tells the nurse, “I‟m constipated all the time!” Which of the following should the nurse recommend? A. Daily enemas B. Laxatives C. Increased fiber intake (your answer) D. Decreased fluid intake Explanation During the third trimester, the enlarging uterus places pressure on the intestines. This coupled with the effect of hormones on smooth muscle relaxation causes decreased intestinal motility (peristalsis). Increasing fiber in the diet will help fecal matter pass more quickly through the intestinal tract, thus decreasing the amount of water that is absorbed. As a result, stool is softer and easier to pass. Enemas could precipitate preterm labor and/or electrolyte loss and should be avoided. Laxatives may cause preterm labor by stimulating peristalsis and may interfere with the absorption of nutrients. Use for more than 1 week can also lead to laxative dependency. Liquid in the diet helps provide a semisolid, soft consistency to the stool. Eight to ten glasses of fluid per day are essential to maintain hydration and promote stool evacuation.

Incorrect Q.35)

44% answered this correctly. 56% answered wrong

The nurse documents positive ballottement in the client‟s prenatal record. The nurse understands that this indicates which of the following? A. Palpable contractions on the abdomen B. Passive movement of the unengaged fetus (correct answer) C. Fetal kicking felt by the client (your answer) D. Enlargement and softening of the uterus Explanation Ballottement indicates passive movement of the unengaged fetus. Ballottement is not a contraction. Fetal kicking felt by the client represents quickening. Enlargement and softening of the uterus is known as Piskacek‟s sign.

Correct Q.36)

79% answered this correctly. 21% answered wrong

The client tells the nurse that her last menstrual period started on January 14 and ended on January 20. Using Nagele‟s rule, the nurse determines her EDD to be which of the following? A. September 27 B. October 21 (your answer) C. November 7 D. December 27 Explanation To calculate the EDD by Nagele‟s rule, add 7 days to the first day of the last menstrual period and count back 3 months, changing the year appropriately. To obtain a date of September 27, 7 days have been added to the last day of the LMP (rather than the first day of the LMP), plus 4 months (instead of 3 months) were counted back. To obtain the date of November 7, 7 days have been subtracted (instead of added) from the first day of LMP plus November indicates counting back 2 months (instead of 3 months) from January. To obtain the date of December 27, 7 days were added to the last day of the LMP (rather than the first day of the LMP) and December indicates counting back only 1 month (instead of 3 months) from January.

Correct Q.37)

71% answered this correctly. 29% answered wrong

The postterm neonate with meconium-stained amniotic fluid needs care designed to especially monitor for which of the following? A. Respiratory problems (your answer) B. Gastrointestinal problems C. Integumentary problems D. Elimination problems Explanation Intrauterine anoxia may cause relaxation of the anal sphincter and emptying of meconium into the amniotic fluid. At birth some of the meconium fluid may be aspirated, causing mechanical obstruction or chemical pneumonitis. The infant is not at increased risk for gastrointestinal problems. Even though the skin is stained with meconium, it is noninfectious (sterile) and nonirritating. The postterm meconiumstained infant is not at additional risk for bowel or urinary problems.

44% answered this correctly. 56% answered wrong

Incorrect Q.38)

When teaching a group of adolescents about variations in the length of the menstrual cycle, the nurse understands that the underlying mechanism is due to variations in which of the following phases? A. Menstrual phase B. Proliferative phase (correct answer) C. Secretory phase (your answer) D. Ischemic phase Explanation Variations in the length of the menstrual cycle are due to variations in the proliferative phase. The menstrual, secretory and ischemic phases do not contribute to this variation. 85% answered this correctly. 15% answered wrong

Correct Q.39)

The amniotic fluid of a client has a greenish tint. The nurse interprets this to be the result of which of the following? A. Lanugo B. Hydramnio C. Meconium (your answer) D. Vernix Explanation The greenish tint is due to the presence of meconium. Lanugo is the soft, downy hair on the shoulders and back of the fetus. Hydramnios represents excessive amniotic fluid. Vernix is the white, cheesy substance covering the fetus. 82% answered this correctly. 18% answered wrong

Correct Q.40)

For the client who is using oral contraceptives, the nurse informs the client about the need to take the pill at the same time each day to accomplish which of the following? A. Decrease the incidence of nausea B. Maintain hormonal levels (your answer) C. Reduce side effects D. Prevent drug interactions Explanation Regular timely ingestion of oral contraceptives is necessary to maintain hormonal levels of the drugs to suppress the action of the hypothalamus and anterior pituitary leading to inappropriate secretion of FSH and LH. Therefore, follicles do not mature, ovulation is inhibited, and pregnancy is prevented. The estrogen content of the oral site contraceptive may cause the nausea, regardless of when the pill is taken. Side effects and drug interactions may occur with oral contraceptives regardless of the time the pill is taken. 66% answered this correctly. 34% answered wrong

Correct Q.41)

A postpartum primipara asks the nurse, “When can we have sexual intercourse again?” Which of the following would be the nurse‟s best response? A. “Anytime you both want to.” B. “As soon as choose a contraceptive method.” C. “When the discharge has stopped and the incision is healed.”(your answer) D. “After your 6 weeks examination.” Explanation Cessation of the lochial discharge signifies healing of the endometrium. Risk of hemorrhage and infection are minimal 3 weeks after a normal vaginal delivery. Telling the client anytime is inappropriate because this response does not provide the client with the specific information she is requesting. Choice of a contraceptive method is important, but not the specific criteria for safe resumption of sexual activity. Culturally, the 6- weeks‟ examination has been used as the time frame for resuming sexual activity, but it may be resumed earlier. 50% answered this correctly. 50% answered wrong

Incorrect Q.42)

When measuring a client‟s fundal height, which of the following techniques denotes the correct method of measurement used by the nurse? A. B. C. D.

From the xiphoid process to the umbilicus (your answer) From the symphysis pubis to the xiphoid process From the symphysis pubis to the fundus (correct answer) From the fundus to the umbilicus

Explanation The nurse should use a nonelastic, flexible, paper measuring tape, placing the zero point on the superior border of the symphysis pubis and stretching the tape across the abdomen at the midline to the top of the fundus. The xiphoid and umbilicus are not appropriate landmarks to use when measuring the height of the fundus (McDonald‟s measurement). Correct Q.43)

56% answered this correctly. 44% answered wrong

The nurse understands that the fetal head is in which of the following positions with a face presentation? A. Completely flexed B. Completely extended (your answer) C. Partially extended D. Partially flexed Explanation With a face presentation, the head is completely extended. With a vertex presentation, the head is completely or partially flexed. With a brow (forehead) presentation, the head would be partially extended.

Correct Q.44)

57% answered this correctly. 43% answered wrong

During the first 4 hours after a male circumcision, assessing for which of the following is the priority? A. Infection B. Hemorrhage (your answer) C. Discomfort D. Dehydration Explanation Hemorrhage is a potential risk following any surgical procedure. Although the infant has been given vitamin K to facilitate clotting, the prophylactic dose is often not sufficient to prevent bleeding. Although infection is a possibility, signs will not appear within 4 hours after the surgical procedure. The primary discomfort of circumcision occurs during the surgical procedure, not afterward. Although feedings are withheld prior to the circumcision, the chances of dehydration are minimal.

Incorrect Q.45)

28% answered this correctly. 72% answered wrong

Immediately after birth the nurse notes the following on a male newborn: respirations 78; apical hearth rate 160 BPM, nostril flaring; mild intercostal retractions; and grunting at the end of expiration. Which of the following should the nurse do? A. Call the assessment data to the physician‟s attention B. Start oxygen per nasal cannula at 2 L/min. C. Suction the infant‟s mouth and nares (your answer) D. Recognize this as normal first period of reactivity (correct answer) Explanation The first 15 minutes to 1 hour after birth is the first period of reactivity involving respiratory and circulatory adaptation to extrauterine life. The data given reflect the normal changes during this time period. The infant‟s assessment data reflect normal adaptation. Thus, the physician does not need to be notified and oxygen is not needed. The data do not indicate the presence of choking, gagging or coughing, which are signs of excessive secretions. Suctioning is not necessary.

Correct Q.46)

42% answered this correctly. 58% answered wrong

When preparing a woman who is 2 days postpartum for discharge, recommendations for which of the following contraceptive methods would be avoided? A. Diaphragm (your answer) B. Female condom C. Oral contraceptives D. Rhythm method Explanation The diaphragm must be fitted individually to ensure effectiveness. Because of the changes to the reproductive structures during pregnancy and following delivery, the diaphragm must be refitted, usually at the 6 weeks‟ examination following childbirth or after a weight loss of 15 lbs or more. In addition, for maximum effectiveness, spermicidal jelly should be placed in the dome and around the rim. However, spermicidal jelly should not be inserted into the vagina until involution is completed at approximately 6 weeks. Use of a female condom protects the reproductive system from the introduction of semen or spermicides into the vagina and may be used after childbirth. Oral contraceptives may be started within the first postpartum week to ensure suppression of ovulation. For the couple who has determined the female‟s fertile period, using the rhythm method, avoidance of intercourse during this period, is safe and effective.

Correct Q.47)

29% answered this correctly. 71% answered wrong

The nurse assesses the vital signs of a client, 4 hours‟ postpartum that are as follows: BP 90/60; temperature 100.4ºF; pulse 100

weak, thready; R 20 per minute. Which of the following should the nurse do first? A. Report the temperature to the physician B. Recheck the blood pressure with another cuff C. Assess the uterus for firmness and position D. Determine the amount of lochia (your answer) Explanation A weak, thready pulse elevated to 100 BPM may indicate impending hemorrhagic shock. An increased pulse is a compensatory mechanism of the body in response to decreased fluid volume. Thus, the nurse should check the amount of lochia present. Temperatures up to 100.48F in the first 24 hours after birth are related to the dehydrating effects of labor and are considered normal. Although rechecking the blood pressure may be a correct choice of action, it is not the first action that should be implemented in light of the other data. The data indicate a potential impending hemorrhage. Assessing the uterus for firmness and position in relation to the umbilicus and midline is important, but the nurse should check the extent of vaginal bleeding first. Then it would be appropriate to check the uterus, which may be a possible cause of the hemorrhage. Incorrect Q.48)

66% answered this correctly. 34% answered wrong

Which of the following would the nurse use as the basis for the teaching plan when caring for a pregnant teenager concerned about gaining too much weight during pregnancy? A. 10 pounds per trimester B. 1 pound per week for 40 weeks C. ½ pound per week for 40 weeks (your answer) D. A total gain of 25 to 30 pounds (correct answer) Explanation To ensure adequate fetal growth and development during the 40 weeks of a pregnancy, a total weight gain 25 to 30 pounds is recommended: 1.5 pounds in the first 10 weeks; 9 pounds by 30 weeks; and 27.5 pounds by 40 weeks. The pregnant woman should gain less weight in the first and second trimester than in the third. During the first trimester, the client should only gain 1.5 pounds in the first 10 weeks, not 1 pound per week. A weight gain of ½ pound per week would be 20 pounds for the total pregnancy, less than the recommended amount.

Correct Q.49)

70% answered this correctly. 30% answered wrong

During a prenatal class, the nurse explains the rationale for breathing techniques during preparation for labor based on the understanding that breathing techniques are most important in achieving which of the following? A. Eliminate pain and give the expectant parents something to do B. Reduce the risk of fetal distress by increasing uteroplacental perfusion C. Facilitate relaxation, possibly reducing the perception of pain(your answer) D. Eliminate pain so that less analgesia and anesthesia are needed Explanation Breathing techniques can raise the pain threshold and reduce the perception of pain. They also promote relaxation. Breathing techniques do not eliminate pain, but they can reduce it. Positioning, not breathing, increases uteroplacental perfusion.

Incorrect Q.50)

45% answered this correctly. 55% answered wrong

Which of the following should the nurse do when a primipara who is lactating tells the nurse that she has sore nipples? A. Tell her to breast feed more frequently (correct answer) B. Administer a narcotic before breast feeding C. Encourage her to wear a nursing brassiere (your answer) D. Use soap and water to clean the nipples Explanation Feeding more frequently, about every 2 hours, will decrease the infant‟s frantic, vigorous sucking from hunger and will decrease breast engorgement, soften the breast, and promote ease of correct latching-on for feeding. Narcotics administered prior to breast feeding are passed through the breast milk to the infant, causing excessive sleepiness. Nipple soreness is not severe enough to warrant narcotic analgesia. All postpartum clients, especially lactating mothers, should wear a supportive brassiere with wide cotton straps. This does not, however, prevent or reduce nipple soreness. Soaps are drying to the skin of the nipples and should not be used on the breasts of lactating mothers. Dry nipple skin predisposes to cracks and fissures, which can become sore and painful.

Question. 1) Correct Terence suffered form burn injury. Using the rule of nines, which has the largest percent of burns?

A. Right thigh and penis B. Right upper arm and penis C. Upper trunk (Your Answer) D. Face and neck Explanation The percentage designated for each burned part of the body using the rule of nines: Head and neck 9%; Right upper extremity 9%; Left upper extremity 9%; Anterior trunk 18%; Posterior trunk 18%; Right lower extremity 18%; Left lower extremity 18%; Perineum 1%. Question. 2) Incorrect Nurse Ron begins to teach a male client how to perform colostomy irrigations. The nurse would evaluate that the instructions were understood when the client states, “I should:

A. Lie on my left side while instilling the irrigating solution.” B. Instill a minimum of 1200 ml of irrigating solution to stimulate evacuation of the bowel.” C. Keep the irrigating container less than 18 inches above the stoma.”(Correct Answer) D. Insert the irrigating catheter deeper into the stoma if cramping occurs during the procedure.” (Your Answer) Explanation This height permits the solution to flow slowly with little force so that excessive peristalsis is not immediately precipitated. Question. 3) Incorrect During vaginal examination of Janah who is in labor, the presenting part is at station plus two. Nurse, correctly interprets it as:

A. Biparietal diameter is 2 cm above the ischial spines. B. Biparietal diameter is at the level of the ischial spines. C. Presenting part is 2 cm above the plane of the ischial spines. (Your Answer) D. Presenting part in 2 cm below the plane of the ischial spines. (Correct Answer) Explanation Fetus at station plus two indicates that the presenting part is 2 cm below the plane of the ischial spines. Question. 4) Incorrect A female client with a fecal impaction frequently exhibits which clinical manifestation? A. Hard, brown, formed stools (Your Answer) B. Loss of urge to defecate C. Increased appetite D. Liquid or semi-liquid stools (Correct Answer) Explanation Passage of liquid or semi-liquid stools results from seepage of unformed bowel contents around the impacted stool in the rectum. Clients with fecal impaction don't pass hard, brown, formed stools because the feces can't move past the impaction. These clients typically report the urge to defecate (although they can't pass stool) and a decreased appetite. Question. 5) Correct Which action should nurse Marian include in the care plan for a 2 month old with heart failure?

A. Weigh and bathe the infant before feeding. B. Allow the infant to rest before feeding. (Your Answer) C. Bathe the infant and administer medications before feeding. D. Feed the infant when he cries. Explanation Because feeding requires so much energy, an infant with heart failure should rest before feeding. Question. 6) Correct A 77-year-old male client is admitted for elective knee surgery. Physical examination reveals shallow respirations but no sign of respiratory distress. Which of the following is a normal physiologic change related to aging?

A. Increased elastic recoil of the lungs

B. Decreased residual volume C. Increased number of functional capillaries in the alveoli D. Decreased vital capacity (Your Answer) Explanation Reduction in vital capacity is a normal physiologic changes include decreased elastic recoil of the lungs, fewer functional capillaries in the alveoli, and an increased in residual volume. Question. 7) Incorrect Tonny has undergoes a left thoracotomy and a partial pneumonectomy. Chest tubes are inserted, and one-bottle water-seal drainage is instituted in the operating room. In the postanesthesia care unit Tonny is placed in Fowler's position on either his right side or on his back. The nurse is aware that this position:

A. Reduce incisional pain. B. Facilitate ventilation of the left lung. (Correct Answer) C. Increase venous return D. Equalize pressure in the pleural space. (Your Answer) Explanation Since only a partial pneumonectomy is done, there is a need to promote expansion of this remaining Left lung by positioning the client on the opposite unoperated side. Question. 8) Incorrect A pregnant client is receiving oxytocin (Pitocin) for induction of labor. A condition that warrant the nurse in-charge to discontinue I.V. infusion of Pitocin is:

A. Fetal heart rate baseline 140-160 bpm. B. Contractions every 1 ½ minutes lasting 70-80 seconds. (Correct Answer) C. Early decelerations in the fetal heart rate. D. Maternal temperature 101.2 (Your Answer) Explanation Contractions every 1 ½ minutes lasting 70-80 seconds, is indicative of hyperstimulation of the uterus, which could result in injury to the mother and the fetus if Pitocin is not discontinued. Question. 9) Correct Nurse Gina is aware that the dietary implications for a client in manic phase of bipolar disorder is:

A. Give the client pieces of cut-up steak, carrots, and an apple. B. Serve the client a bowl of soup, buttered French bread, and apple slices. C. Increase calories, decrease fat, and decrease protein. D. Increase calories, carbohydrates, and protein. (Your Answer) Explanation This client increased protein for tissue building and increased calories to replace what is burned up (usually via carbohydrates). Question. 10) Incorrect Dervid, an adolescent has a history of truancy from school, running away from home and “barrowing” other people‟s things without their permission. The adolescent denies stealing, rationalizing instead that as long as no one was using the items, it was all right to borrow them. It is important for the nurse to understand the psychodynamically, this behavior may be largely attributed to a developmental defect related to the: A. Id (Your Answer) B. Superego (Correct Answer) C. Ego D. Oedipal complex Explanation This behavior shows a weak sense of moral consciousness. According to Freudian theory, personality disorders stem from a weak superego. Question. 11) Correct A trial for vaginal delivery after an earlier caesareans, would likely to be given to a gravida, who had:

A.

First and second caesareans were for cephalopelvic disproportion.

B. First caesarean through a classic incision as a result of severe fetal distress. C. First low transverse cesarean was for active herpes type 2 infections; vaginal culture at 39 weeks pregnancy was positive. D. First low transverse caesarean was for breech position. Fetus in this pregnancy is in a vertex presentation. (Your Answer) Explanation This type of client has no obstetrical indication for a caesarean section as she did with her first caesarean delivery. Question. 12) Correct A male client has active tuberculosis (TB). Which of the following symptoms will be exhibit?

A. Fever of more than 104°F (40°C) and nausea B. Chills, fever, night sweats, and hemoptysis (Your Answer) C. Chest and lower back pain D. Headache and photophobia Explanation Typical signs and symptoms are chills, fever, night sweats, and hemoptysis. Chest pain may be present from coughing, but isn‟t usual. Clients with TB typically have low-grade fevers, not higher than 102°F (38.9°C). Nausea, headache, and photophobia aren‟t usual TB symptoms. Question. 13) Correct Nurse Linda prepares to perform an otoscopic examination on a female client. For proper visualization, the nurse should position the client's ear by: A. Pulling the lobule down and back B. Pulling the lobule down and forward C. Pulling the helix up and forward D. Pulling the helix up and back (Your Answer) Explanation To perform an otoscopic examination on an adult, the nurse grasps the helix of the ear and pulls it up and back to straighten the ear canal. For a child, the nurse grasps the helix and pulls it down to straighten the ear canal. Pulling the lobule in any direction wouldn't straighten the ear canal for visualization. Question. 14) Correct Baby Tina a 3 month old infant just had a cleft lip and palate repair. What should the nurse do to prevent trauma to operative site?

A. Place the infant‟s arms in soft elbow restraints. (Your Answer) B. Avoid touching the suture line, even when cleaning. C. Give the baby a pacifier. D. Place the baby in prone position. Explanation Soft restraints from the upper arm to the wrist prevent the infant from touching her lip but allow him to hold a favorite item such as a blanket. Because they could damage the operative site, such as objects as pacifiers, suction catheters, and small spoons shouldn‟t be placed in a baby‟s mouth after cleft repair. A baby in a prone position may rub her face on the sheets and traumatize the operative site. The suture line should be cleaned gently to prevent infection, which could interfere with healing and damage the cosmetic appearance of the repair. Question. 15) Correct Tracy is receiving combination chemotherapy for treatment of metastatic carcinoma. Nurse Ruby should monitor the client for the systemic side effect of:

A. Polycythemia B. Nystagmus C. Leukopenia (Your Answer) D. Ascites Explanation Leukopenia, a reduction in WBCs, is a systemic effect of chemotherapy as a result of myelosuppression. Question. 16) Correct Nurse Monett is caring for a client recovering from gastro-intestinal bleeding. The nurse should:

A. Monitor vital signs every 2 hours. B. Make sure that the client takes food and medications at prescribed intervals. (Your Answer) C. Provide milk every 2 to 3 hours. D. Plan care so the client can receive 8 hours of uninterrupted sleep each night. Explanation Food and drug therapy will prevent the accumulation of hydrochloric acid, or will neutralize and buffer the acid that does accumulate. Question. 17) Correct Nurse Gail places a client in a four-point restraint following orders from the physician. The client care plan should include: A. Check circulation every 15-30 minutes. (Your Answer) B. Provide diversional activities. C. Assess temperature frequently. D. Socialize with other patients once a shift. Explanation Restraints encircle the limbs, which place the client at risk for circulation being restricted to the distal areas of the extremities. Checking the client‟s circulation every 15-30 minutes will allow the nurse to adjust the restraints before injury from decreased blood flow occurs. Question. 18) Correct The physician prescribes a loop diuretic for a client. When administering this drug, the nurse anticipates that the client may develop which electrolyte imbalance? A. Hypervolemia B. Hyperkalemia C. Hypernatremia D. Hypokalemia (Your Answer) Explanation A loop diuretic removes water and, along with it, sodium and potassium. This may result in hypokalemia, hypovolemia, and hyponatremia. Question. 19) Correct Herbert, a 45 year old construction engineer is brought to the hospital unconscious after falling from a 2-story building. When assessing the client, the nurse would be most concerned if the assessment revealed:

A. Bleeding from ears (Your Answer) B. Reactive pupils C. A depressed fontanel D. An elevated temperature Explanation The nurse needs to perform a thorough assessment that could indicate alterations in cerebral function, increased intracranial pressures, fractures and bleeding. Bleeding from the ears occurs only with basal skull fractures that can easily contribute to increased intracranial pressure and brain herniation. Question. 20) Correct Mario has burn injury. After Forty48 hours, the physician orders for Mario 2 liters of IV fluid to be administered q12 h. The drop factor of the tubing is 10 gtt/ml. The nurse should set the flow to provide:

A. 32 gtt/min B. 36 gtt/min C. 28 gtt/min (Your Answer) D. 18 gtt/min Explanation This is the correct flow rate; multiply the amount to be infused (2000 ml) by the drop factor (10) and divide the result by the amount of time in minutes (12 hours x 60 minutes) Question. 21) Correct The nurse is ware that the most relevant knowledge about oxygen administration to a male client with COPD is

A. B.

Oxygen is administered best using a non-rebreathing mask Hypoxia stimulates the central chemoreceptors in the medulla that makes the client breath.

C. Blood gases are monitored using a pulse oximeter. D. Oxygen at 1-2L/min is given to maintain the hypoxic stimulus for breathing.(Your Answer) Explanation COPD causes a chronic CO2 retention that renders the medulla insensitive to the CO2 stimulation for breathing. The hypoxic state of the client then becomes the stimulus for breathing. Giving the client oxygen in low concentrations will maintain the client‟s hypoxic drive. Question. 22) Incorrect A client undergone spinal anesthetic, it will be important that the nurse immediately position the client in:

A. Flat on the stomach, with the head turned to the side. B. On the side, to prevent obstruction of airway by tongue. (Your Answer) C. Flat on back. (Correct Answer) D. On the back, with knees flexed 15 degrees. Explanation To avoid the complication of a painful spinal headache that can last for several days, the client is kept in flat in a supine position for approximately 4 to 12 hours postoperatively. Headaches are believed to be causes by the seepage of cerebral spinal fluid from the puncture site. By keeping the client flat, cerebral spinal fluid pressures are equalized, which avoids trauma to the neurons. Question. 23) Correct The doctor orders hourly urine output measurement for a postoperative male client. The nurse Trish records the following amounts of output for 2 consecutive hours: 8 a.m.: 50 ml; 9 a.m.: 60 ml. Based on these amounts, which action should the nurse take? A. Irrigate the indwelling urinary catheter B. Continue to monitor and record hourly urine output (Your Answer) C. Notify the physician D. Increase the I.V. fluid infusion rate Explanation Normal urine output for an adult is approximately 1 ml/minute (60 ml/hour). Therefore, this client's output is normal. Beyond continued evaluation, no nursing action is warranted. Question. 24) Correct Tony, a basketball player twist his right ankle while playing on the court and seeks care for ankle pain and swelling. After the nurse applies ice to the ankle for 30 minutes, which statement by Tony suggests that ice application has been effective? A. “My ankle appears redder now”. B. “My ankle feels warm”. C. “My ankle looks less swollen now”. (Your Answer) D. “I need something stronger for pain relief” Explanation Ice application decreases pain and swelling. Continued or increased pain, redness, and increased warmth are signs of inflammation that shouldn't occur after ice application Question. 25) Incorrect Mrs. Cruz, 80 years old is diagnosed with pneumonia. Which of the following symptoms may appear first?

A. Altered mental status and dehydration (Correct Answer) B. Pleuritic chest pain and cough (Your Answer) C. Hemoptysis and Dyspnea D. Fever and chills Explanation Fever, chills, hemortysis, dyspnea, cough, and pleuritic chest pain are the common symptoms of pneumonia, but elderly clients may first appear with only an altered lentil status and dehydration due to a blunted immune response. Question. 26) Correct Calcium gluconate is being administered to a client with pregnancy induced hypertension (PIH). A nursing action that must be initiated as the plan of care throughout injection of the drug is:

A. Continuous CPR

B. CVP readings C. EKG tracings (Your Answer) D. Ventilator assistance Explanation A potential side effect of calcium gluconate administration is cardiac arrest. Continuous monitoring of cardiac activity (EKG) throught administration of calcium gluconate is an essential part of care. Question. 27) Correct A male client who has severe burns is receiving H2 receptor antagonist therapy. The nurse In-charge knows the purpose of this therapy is to: A. Facilitate protein synthesis. B. Prevent stress ulcer (Your Answer) C. Block prostaglandin synthesis D. Enhance gas exchange Explanation Curling‟s ulcer occurs as a generalized stress response in burn patients. This results in a decreased production of mucus and increased secretion of gastric acid. The best treatment for this prophylactic use of antacids and H2 receptor blockers Question. 28) Correct Nurse Michelle is assessing a 24 year old client with a diagnosis of hydatidiform mole. She is aware that one of the following is unassociated with this condition?

A. Excessive fetal activity. (Your Answer) B. Vaginal bleeding C. Larger than normal uterus for gestational age. D. Elevated levels of human chorionic gonadotropin. Explanation The most common signs and symptoms of hydatidiform mole includes elevated levels of human chorionic gonadotropin, vaginal bleeding, larger than normal uterus for gestational age, failure to detect fetal heart activity even with sensitive instruments, excessive nausea and vomiting, and early development of pregnancy-induced hypertension. Fetal activity would not be noted. Question. 29) Incorrect What is Nurse John likely to note in a male client being admitted for alcohol withdrawal?

A. Impending coma. B. Depression with mutism. C. Perceptual disorders. (Correct Answer) D. Recent alcohol intake. (Your Answer) Explanation Frightening visual hallucinations are especially common in clients experiencing alcohol withdrawal. Question. 30) Incorrect Patrick is in the oliguric phase of acute tubular necrosis and is experiencing fluid and electrolyte imbalances. The client is somewhat confused and complains of nausea and muscle weakness. As part of the prescribed therapy to correct this electrolyte imbalance, the nurse would expect to:

A. Administer Kayexalate (Correct Answer) B. Restrict foods high in protein C. Increase oral intake of cheese and milk. (Your Answer) D. Administer large amounts of normal saline via I.V. Explanation Kayexalate,a potassium exchange resin, permits sodium to be exchanged for potassium in the intestine, reducing the serum potassium level. Question. 31) Incorrect Nurse Sherry is teaching male client regarding his permanent artificial pacemaker. Which information given by the nurse shows her knowledge deficit about the artificial cardiac pacemaker?

A.

May be allowed to use electrical appliances (Your Answer)

B. Have regular follow up care C. take the pulse rate once a day, in the morning upon awakening D. May engage in contact sports (Correct Answer) Explanation The client should be advised by the nurse to avoid contact sports. This will prevent trauma to the area of the pacemaker generator. Question. 32) Correct A client undergone ileostomy, when should the drainage appliance be applied to the stoma?

A. 24 hours later, when edema has subsided. B. When the client is able to begin self-care procedures. C. In the operating room. (Your Answer) D. After the ileostomy begin to function. Explanation The stoma drainage bag is applied in the operating room. Drainage from the ileostomy contains secretions that are rich in digestive enzymes and highly irritating to the skin. Protection of the skin from the effects of these enzymes is begun at once. Skin exposed to these enzymes even for a short time becomes reddened, painful, and excoriated. Question. 33) Incorrect Nurse Patricia finds a female client who is post-myocardial infarction (MI) slumped on the side rails of the bed and unresponsive to shaking or shouting. Which is the nurse next action?

A. Give two sharp thumps to the precordium, and check the pulse. (Your Answer) B. Call for help and note the time. (Correct Answer) C. Administer two quick blows. D. Clear the airway Explanation Having established, by stimulating the client, that the client is unconscious rather than sleep, the nurse should immediately call for help. This may be done by dialing the operator from the client‟s phone and giving the hospital code for cardiac arrest and the client‟s room number to the operator, of if the phone is not available, by pulling the emergency call button. Noting the time is important baseline information for cardiac arrest procedure. Question. 34) Correct Nurse Ryan is aware that the best initial approach when trying to take a crying toddler‟s temperature is:

A. Talk to the mother first and then to the toddler. (Your Answer) B. Encourage the mother to hold the child. C. Bring extra help so it can be done quickly. D. Ignore the crying and screaming. Explanation When dealing with a crying toddler, the best approach is to talk to the mother and ignore the toddler first. This approach helps the toddler get used to the nurse before she attempts any procedures. It also gives the toddler an opportunity to see that the mother trusts the nurse. Question. 35) Correct Norma, with recent colostomy expresses concern about the inability to control the passage of gas. Nurse Oliver should suggest that the client plan to:

A. Decrease fluid intake at meal times. B. Adhere to a bland diet prior to social events. C. Eliminate foods high in cellulose. D. Avoid foods that in the past caused flatus. (Your Answer) Explanation Foods that bothered a person preoperatively will continue to do so after a colostomy. Question. 36) Incorrect In preparing a female client for electroconvulsive therapy (ECT), Nurse Michelle knows that succinylcoline (Anectine) will be administered for which therapeutic effect?

A. Skeletal muscle paralysis. (Correct Answer) B. Decreased oral and respiratory secretions. (Your Answer) C. Short-acting anesthesia D. Analgesia. Explanation Anectine is a depolarizing muscle relaxant causing paralysis. It is used to reduce the intensity of muscle contractions during the convulsive stage, thereby reducing the risk of bone fractures or dislocation. Question. 37) Incorrect A male client was on warfarin (Coumadin) before admission, and has been receiving heparin I.V. for 2 days. The partial thromboplastin time (PTT) is 68 seconds. What should Nurse Carla do?

A. Expect the warfarin to increase the PTT. B. Continue treatment as ordered. (Correct Answer) C. Increase the dosage, because the level is lower than normal. D. Stop the I.V. infusion of heparin and notify the physician. (Your Answer) Explanation The effects of heparin are monitored by the PTT is normally 30 to 45 seconds; the therapeutic level is 1.5 to 2 times the normal level. Question. 38) Incorrect Which type of medication order might read "Vitamin K 10 mg I.M. daily m 3 days?" A. Standard written order (Correct Answer) B. Single order C. Standing order (Your Answer) D. Stat order Explanation This is a standard written order. Prescribers write a single order for medications given only once. A stat order is written for medications given immediately for an urgent client problem. A standing order, also known as a protocol, establishes guidelines for treating a particular disease or set of symptoms in special care areas such as the coronary care unit. Facilities also may institute medication protocols that specifically designate drugs that a nurse may not give. Question. 39) Correct Marichu was given morphine sulfate for pain. She is sleeping and her respiratory rate is 4 breaths/minute. If action isn‟t taken quickly, she might have which of the following reactions?

A. Seizure B. Asthma attack C. Wake up on his own D. Respiratory arrest (Your Answer) Explanation Narcotics can cause respiratory arrest if given in large quantities. It‟s unlikely the client will have asthma attack or a seizure or wake up on his own. Question. 40) Correct What parental behavior toward a child during an admission procedure should cause Nurse Ron to suspect child abuse?

A. Expressing guilt B. Acting overly solicitous toward the child. (Your Answer) C. Ignoring the child. D. Flat affect Explanation This behavior is an example of reaction formation, a coping mechanism. Question. 41) Incorrect Nurse Hazel is teaching a mother who plans to discontinue breast feeding after 5 months. The nurse should advise her to include which foods in her infant‟s diet?

A. B.

Iron-rich formula only. (Correct Answer) Skim milk and baby food.

C. Iron-rich formula and baby food. (Your Answer) D. Whole milk and baby food. Explanation The infants at age 5 months should receive iron-rich formula and that they shouldn‟t receive solid food, even baby food until age 6 months. Question. 42) Incorrect Nurse Amy is aware that the following is true about functional nursing

A. Concentrates on tasks and activities. (Your Answer) B. Emphasize the use of group collaboration. C. Provides continuous, coordinated and comprehensive nursing services.(Correct Answer) D. One-to-one nurse patient ratio. Explanation Functional nursing is focused on tasks and activities and not on the care of the patients. Question. 43) Incorrect After seeking help at an outpatient mental health clinic, Ruby who was raped while walking her dog is diagnosed with posttraumatic stress disorder (PTSD). Three months later, Ruby returns to the clinic, complaining of fear, loss of control, and helpless feelings. Which nursing intervention is most appropriate for Ruby? A. Allowing the client time to heal. B. Exploring the meaning of the traumatic event with the client. (Correct Answer) C. Giving sleep medication, as prescribed, to restore a normal sleepwake cycle. (Your Answer) D. Recommending a high-protein, low-fat diet. Explanation The client with PTSD needs encouragement to examine and understand the meaning of the traumatic event and consequent losses. Otherwise, symptoms may worsen and the client may become depressed or engage in self-destructive behavior such as substance abuse. The client must explore the meaning of the event and won't heal without this, no matter how much time passes. Behavioral techniques, such as relaxation therapy, may help decrease the client's anxiety and induce sleep. The physician may prescribe antianxiety agents or antidepressants cautiously to avoid dependence; sleep medication is rarely appropriate. A special diet isn't indicated unless the client also has an eating disorder or a nutritional problem. Question. 44) Incorrect Nurse Lynnette notices that a female client with obsessive-compulsive disorder washes her hands for long periods each day. How should the nurse respond to this compulsive behavior?

A. By calling attention to or attempting to prevent the behavior. B. By designating times during which the client can focus on the behavior.(Correct Answer) C. By urging the client to reduce the frequency of the behavior as rapidly as possible. (Your Answer) D. By discouraging the client from verbalizing anxieties. Explanation The nurse should designate times during which the client can focus on the compulsive behavior or obsessive thoughts. The nurse should urge the client to reduce the frequency of the compulsive behavior gradually, not rapidly. She shouldn't call attention to or try to prevent the behavior. Trying to prevent the behavior may cause pain and terror in the client. The nurse should encourage the client to verbalize anxieties to help distract attention from the compulsive behavior. Question. 45) Incorrect Nurse Betty is assigned to the following clients. The client that the nurse would see first after endorsement? A 34 year-old post operative appendectomy client of five hours who is complaining of pain. A 26 year-old client admitted for dehydration whose intravenous (IV) has infiltrated. (Your Answer) A 44 year-old myocardial infarction (MI) client who is complaining of nausea. (Correct Answer) A 63 year-old post operative‟s abdominal hysterectomy client of three days whose incisional dressing is saturated with serosanguinous fluid. Explanation Nausea is a symptom of impending myocardial infarction (MI) and should be assessed immediately so that treatment can be instituted and further damage to the heart is avoided. A. B. C. D.

Question. 46) Incorrect Mark, a 7-year-old client is brought to the emergency department. He‟s tachypneic and afebrile and has a respiratory rate of 36 breaths/minute and has a nonproductive cough. He recently had a cold. Form this history; the client may have which of the following conditions?

A. Emphysema B. Acute asthma (Correct Answer) C. Chronic obstructive pulmonary disease (COPD) D. Bronchial pneumonia (Your Answer) Explanation Based on the client‟s history and symptoms, acute asthma is the most likely diagnosis. He‟s unlikely to have bronchial pneumonia without a productive cough and fever and he‟s too young to have developed (COPD) and emphysema. Question. 47) Correct A pregnant client is receiving magnesium sulfate for severe pregnancy induced hypertension (PIH). The clinical findings that would warrant use of the antidote , calcium gluconate is:

A. Rapid rise in blood pressure. B. Rapid respiratory rate above 40/min. C. Urinary output 90 cc in 2 hours. D. Absent patellar reflexes. (Your Answer) Explanation Absence of patellar reflexes is an indicator of hypermagnesemia, which requires administration of calcium gluconate. Question. 48) Correct Aira has taken amitriptyline HCL (Elavil) for 3 days, but now complains that it “doesn‟t help” and refuses to take it. What should the nurse say or do? A. Withhold the drug. B. Suggest that it takes awhile before seeing the results. (Your Answer) C. Encourage the client to tell the doctor. D. Record the client‟s response. Explanation The client needs a specific response; that it takes 2 to 3 weeks (a delayed effect) until the therapeutic blood level is reached. Question. 49) Correct She finds out that some managers have benevolent-authoritative style of management. Which of the following behaviors will she exhibit most likely? A. Communicates downward to staffs. B. Gives economic and ego awards. C. Have condescending trust and confidence in their subordinates. (Your Answer) D. Allows decision making among subordinates. Explanation Benevolent-authoritative managers pretentiously show their trust and confidence to their followers. Question. 50) Incorrect While monitoring a male client several hours after a motor vehicle accident, which assessment data suggest increasing intracranial pressure?

A. Blood pressure is decreased from 160/90 to 110/70. B. The client refuses dinner because of anorexia. (Your Answer) C. Pulse is increased from 87 to 95, with an occasional skipped beat. D. The client is oriented when aroused from sleep, and goes back to sleep immediately. (Correct Answer) Explanation This finding suggest that the level of consciousness is decreasing.

36% answered this correctly. 64% answered wrong

Incorrect Q.1)

Nurse Oliver is teaching a diabetic pregnant client about nutrition and insulin needs during pregnancy. The nurse determines that the client understands dietary and insulin needs if the client states that the second half of pregnancy require: A. B. C. D.

Decreased caloric intake Increased caloric intake (correct answer) Decreased Insulin (your answer) Increase Insulin

Explanation Glucose crosses the placenta, but insulin does not. High fetal demands for glucose, combined with the insulin resistance caused by hormonal changes in the last half of pregnancy can result in elevation of maternal blood glucose levels. This increases the mother‟s demand for insulin and is referred to as the diabetogenic effect of pregnancy. Incorrect Q.2)

21% answered this correctly. 79% answered wrong

Maylene had just received her 4th dose of tetanus toxoid. She is aware that her baby will have protection against tetanus for A. 1 year (correct answer) B. 3 years C. 5 years (your answer) D. Lifetime Explanation The baby will have passive natural immunity by placental transfer of antibodies. The mother will have active artificial immunity lasting for about 10 years. 5 doses will give the mother lifetime protection.

Incorrect Q.3)

28% answered this correctly. 72% answered wrong

Myrna a public health nurse will conduct outreach immunization in a barangay Masay with a population of about 1500. The estimated number of infants in the barangay would be: A. 45 infants (correct answer) B. 50 infants C. 55 infants (your answer) D. 65 infants Explanation To estimate the number of infants, multiply total population by 3%.

Correct Q.4)

59% answered this correctly. 41% answered wrong

Tertiary prevention is needed in which stage of the natural history of disease? A. Pre-pathogenesis B. Pathogenesis C. Prodromal D. Terminal (your answer) Explanation Tertiary prevention involves rehabilitation, prevention of permanent disability and disability limitation appropriate for convalescents, the disabled, complicated cases and the terminally ill (those in the terminal stage of a disease).

Incorrect Q.5)

49% answered this correctly. 51% answered wrong

Nurse Hazel is teaching a mother who plans to discontinue breast feeding after 5 months. The nurse should advise her to include which foods in her infant‟s diet? A. Skim milk and baby food. B. Whole milk and baby food. C. Iron-rich formula only. (correct answer) D. Iron-rich formula and baby food. (your answer) Explanation The infants at age 5 months should receive iron-rich formula and that they shouldn‟t receive solid food, even baby food until age 6 months.

Incorrect Q.6)

37% answered this correctly. 63% answered wrong

It is the most effective way of controlling schistosomiasis in an endemic area? A. Use of molluscicides B. Building of foot bridges (your answer) C. Proper use of sanitary toilets (correct answer) D. Use of protective footwear, such as rubber boots Explanation The ova of the parasite get out of the human body together with feces. Cutting the cycle at this stage is the most effective way of preventing the spread of the disease to susceptible hosts.

Correct Q.7)

52% answered this correctly. 48% answered wrong

Which of the following would be least likely to indicate anticipated bonding behaviors by new parents?

A. The parents‟ willingness to touch and hold the new born. B. The parent‟s expression of interest about the size of the new born. C. The parents‟ indication that they want to see the newborn. D. The parents‟ interactions with each other. (your answer) Explanation Parental interaction will provide the nurse with a good assessment of the stability of the family's home life but it has no indication for parental bonding. Willingness to touch and hold the newborn, expressing interest about the newborn's size, and indicating a desire to see the newborn are behaviors indicating parental bonding. Incorrect Q.8)

18% answered this correctly. 82% answered wrong

Nurse John is performing an assessment on a neonate. Which of the following findings is considered common in the healthy neonate? A. Simian crease B. Conjunctival hemorrhage (correct answer) C. Cystic hygroma D. Bulging fontanelle (your answer) Explanation Conjunctival hemorrhages are commonly seen in neonates secondary to the cranial pressure applied during the birth process. Bulging fontanelles are a sign of intracranial pressure. Simian creases are present in 40% of the neonates with trisomy 21. Cystic hygroma is a neck mass that can affect the airway.

Correct Q.9)

58% answered this correctly. 42% answered wrong

The nurse explains to a breastfeeding mother that breast milk is sufficient for all of the baby‟s nutrient needs only up to: A. 5 months B. 6 months (your answer) C. 1 year D. 2 years Explanation After 6 months, the baby‟s nutrient needs, especially the baby‟s iron requirement, can no longer be provided by mother‟s milk alone.

Correct Q.10)

63% answered this correctly. 37% answered wrong

When teaching parents of a neonate the proper position for the neonate‟s sleep, the nurse Patricia stresses the importance of placing the neonate on his back to reduce the risk of which of the following? A. Aspiration B. Sudden infant death syndrome (SIDS) (your answer) C. Suffocation D. Gastroesophageal reflux (GER) Explanation Supine positioning is recommended to reduce the risk of SIDS in infancy. The risk of aspiration is slightly increased with the supine position. Suffocation would be less likely with an infant supine than prone and the position for GER requires the head of the bed to be elevated.

Correct Q.11)

70% answered this correctly. 30% answered wrong

Calcium gluconate is being administered to a client with pregnancy induced hypertension (PIH). A nursing action that must be initiated as the plan of care throughout injection of the drug is: A. Ventilator assistance B. CVP readings C. EKG tracings (your answer) D. Continuous CPR Explanation A potential side effect of calcium gluconate administration is cardiac arrest. Continuous monitoring of cardiac activity (EKG) throught administration of calcium gluconate is an essential part of care.

Incorrect Q.12)

64% answered this correctly. 36% answered wrong

Several clients is newly admitted and diagnosed with leprosy. Which of the following clients should be classified as a case of multibacillary leprosy?

A. 3 skin lesions, negative slit skin smear B. 3 skin lesions, positive slit skin smear (your answer) C. 5 skin lesions, negative slit skin smear D. 5 skin lesions, positive slit skin smear (correct answer) Explanation A multibacillary leprosy case is one who has a positive slit skin smear and at least 5 skin lesions. Incorrect Q.13)

26% answered this correctly. 74% answered wrong

According to Freeman and Heinrich, community health nursing is a developmental service. Which of the following best illustrates this statement? A. The community health nurse continuously develops himself personally and professionally. B. Health education and community organizing are necessary in providing community health services. (correct answer) C. Community health nursing is intended primarily for health promotion and prevention and treatment of disease. (your answer) D. The goal of community health nursing is to provide nursing services to people in their own places of residence. Explanation The community health nurse develops the health capability of people through health education and community organizing activities.

Correct Q.14)

52% answered this correctly. 48% answered wrong

Dianne, 24 year-old is 27 weeks‟ pregnant arrives at her physician‟s office with complaints of fever, nausea, vomiting, malaise, unilateral flank pain, and costovertebral angle tenderness. Which of the following diagnoses is most likely? A. Asymptomatic bacteriuria B. Bacterial vaginosis C. Pyelonephritis (your answer) D. Urinary tract infection (UTI) Explanation The symptoms indicate acute pyelonephritis, a serious condition in a pregnant client. UTI symptoms include dysuria, urgency, frequency, and suprapubic tenderness. Asymptomatic bacteriuria doesn‟t cause symptoms. Bacterial vaginosis causes milky white vaginal discharge but no systemic symptoms.

Correct Q.15)

72% answered this correctly. 28% answered wrong

Baby Tina a 3 month old infant just had a cleft lip and palate repair. What should the nurse do to prevent trauma to operative site? A. Avoid touching the suture line, even when cleaning. B. Place the baby in prone position. C. Give the baby a pacifier. D. Place the infant‟s arms in soft elbow restraints. (your answer) Explanation Soft restraints from the upper arm to the wrist prevent the infant from touching her lip but allow him to hold a favorite item such as a blanket. Because they could damage the operative site, such as objects as pacifiers, suction catheters, and small spoons shouldn‟t be placed in a baby‟s mouth after cleft repair. A baby in a prone position may rub her face on the sheets and traumatize the operative site. The suture line should be cleaned gently to prevent infection, which could interfere with healing and damage the cosmetic appearance of the repair.

Correct Q.16)

73% answered this correctly. 27% answered wrong

A young child named Richard is suspected of having pinworms. The community nurse collects a stool specimen to confirm the diagnosis. The nurse should schedule the collection of this specimen for: A. Just before bedtime B. After the child has been bathe C. Any time during the day D. Early in the morning (your answer) Explanation Based on the nurse‟s knowledge of microbiology, the specimen should be collected early in the morning. The rationale for this timing is that, because the female worm lays eggs at night around the perineal area, the first bowel movement of the day will yield the best results. The specific type of stool specimen used in the diagnosis of pinworms is called the tape test.

Incorrect Q.17)

12% answered this correctly. 88% answered wrong

Beth a public health nurse takes an active role in community participation. What is the primary goal of community organizing? A. To educate the people regarding community health problems

B. To mobilize the people to resolve community health problems(your answer) C. To maximize the community‟s resources in dealing with health problems. D. To maximize the community‟s resources in dealing with health problems. (correct answer) Explanation Community organizing is a developmental service, with the goal of developing the people‟s self-reliance in dealing with community health problems. A, B and C are objectives of contributory objectives to this goal. Incorrect Q.18)

47% answered this correctly. 53% answered wrong

Nurse Hazel is preparing to care for a client who is newly admitted to the hospital with a possible diagnosis of ectopic pregnancy. Nurse Hazel develops a plan of care for the client and determines that which of the following nursing actions is the priority? A. Monitoring weight B. Assessing for edema (your answer) C. Monitoring apical pulse (correct answer) D. Monitoring temperature Explanation Nursing care for the client with a possible ectopic pregnancy is focused on preventing or identifying hypovolemic shock and controlling pain. An elevated pulse rate is an indicator of shock.

Correct Q.19)

34% answered this correctly. 66% answered wrong

Mommy Linda is playing with her infant, who is sitting securely alone on the floor of the clinic. The mother hides a toy behind her back and the infant looks for it. The nurse is aware that estimated age of the infant would be: A. 6 months B. 4 months C. 8 months D. 10 months (your answer) Explanation A 10 month old infant can sit alone and understands object permanence, so he would look for the hidden toy. At age 4 to 6 months, infants can‟t sit securely alone. At age 8 months, infants can sit securely alone but cannot understand the permanence of objects.

Incorrect Q.20)

36% answered this correctly. 64% answered wrong

Nurse Tina is aware that the disease declared through Presidential Proclamation No. 4 as a target for eradication in the Philippines is? A. Poliomyelitis (your answer) B. Measles (correct answer) C. Rabies D. Neonatal tetanus Explanation Presidential Proclamation No. 4 is on the Ligtas Tigdas Program.

Incorrect Q.21)

45% answered this correctly. 55% answered wrong

The skin in the diaper area of a 7 month old infant is excoriated and red. Nurse Hazel should instruct the mother to: A. Change the diaper more often. (correct answer) B. Apply talc powder with diaper changes. (your answer) C. Wash the area vigorously with each diaper change. D. Decrease the infant‟s fluid intake to decrease saturating diapers. Explanation Decreasing the amount of time the skin comes contact with wet soiled diapers will help heal the irritation.

Incorrect Q.22)

38% answered this correctly. 62% answered wrong

Nurses are aware that diagnosis of leprosy is highly dependent on recognition of symptoms. Which of the following is an early sign of leprosy? A. Macular lesions (your answer) B. Inability to close eyelids C. Thickened painful nerves (correct answer) D. Sinking of the nosebridge Explanation

The lesion of leprosy is not macular. It is characterized by a change in skin color (either reddish or whitish) and loss of sensation, sweating and hair growth over the lesion. Inability to close the eyelids (lagophthalmos) and sinking of the nosebridge are late symptoms. Correct Q.23)

77% answered this correctly. 23% answered wrong

Maureen in her third trimester arrives at the emergency room with painless vaginal bleeding. Which of the following conditions is suspected? A. Placenta previa (your answer) B. Abruptio placentae C. Premature labor D. Sexually transmitted disease Explanation Placenta previa with painless vaginal bleeding.

Incorrect Q.24)

47% answered this correctly. 53% answered wrong

Myra is the public health nurse in a municipality with a total population of about 20,000. There are 3 rural health midwives among the RHU personnel. How many more midwife items will the RHU need? A. 1 (correct answer) B. 2 C. 3 D. The RHU does not need any more midwife item. (your answer) Explanation Each rural health midwife is given a population assignment of about 5,000.

Correct Q.25)

63% answered this correctly. 37% answered wrong

Nikki a 5-month old infant was brought by his mother to the health center because of diarrhea for 4 to 5 times a day. Her skin goes back slowly after a skin pinch and her eyes are sunken. Using the IMCI guidelines, you will classify this infant in which category? A. No signs of dehydration B. Some dehydration (your answer) C. Severe dehydration D. The data is insufficient. Explanation Using the assessment guidelines of IMCI, a child (2 months to 5 years old) with diarrhea is classified as having SOME DEHYDRATION if he shows 2 or more of the following signs: restless or irritable, sunken eyes, the skin goes back slow after a skin pinch.

Incorrect Q.26)

24% answered this correctly. 76% answered wrong

Nurse Carla knows that the common cardiac anomalies in children with Down Syndrome (tri-somy 21) is: A. Atrial septal defect B. Pulmonic stenosis C. Ventricular septal defect (your answer) D. Endocardial cushion defect (correct answer) Explanation Endocardial cushion defects are seen most in children with Down syndrome, asplenia, or polysplenia.

Incorrect Q.27)

38% answered this correctly. 62% answered wrong

When a client states that her "water broke," which of the following actions would be inappropriate for the nurse to do? A. Observing the pooling of straw-colored fluid. B. Checking vaginal discharge with nitrazine paper. (your answer) C. Conducting a bedside ultrasound for an amniotic fluid index.(correct answer) D. Observing for flakes of vernix in the vaginal discharge. Explanation It isn't within a nurse's scope of practice to perform and interpret a bedside ultrasound under these conditions and without specialized training. Observing for pooling of straw-colored fluid, checking vaginal discharge with nitrazine paper, and observing for flakes of vernix are appropriate assessments for determining whether a client has ruptured membranes.

Correct

41% answered this correctly. 59% answered wrong

Q.28)

Malou was diagnosed with severe preeclampsia is now receiving I.V. magnesium sulfate. The adverse effects associated with magnesium sulfate is: A. Anemia B. Decreased urine output (your answer) C. Hyperreflexia D. Increased respiratory rate Explanation Decreased urine output may occur in clients receiving I.V. magnesium and should be monitored closely to keep urine output at greater than 30 ml/hour, because magnesium is excreted through the kidneys and can easily accumulate to toxic levels.

Correct Q.29)

56% answered this correctly. 44% answered wrong

May knows that the step in community organizing that involves training of potential leaders in the community is: A. Integration B. Community organization C. Community study D. Core group formation (your answer) Explanation In core group formation, the nurse is able to transfer the technology of community organizing to the potential or informal community leaders through a training program.

Incorrect Q.30)

61% answered this correctly. 39% answered wrong

A pregnant client is receiving oxytocin (Pitocin) for induction of labor. A condition that warrant the nurse in-charge to discontinue I.V. infusion of Pitocin is: A. Contractions every 1 ½ minutes lasting 70-80 seconds.(correct answer) B. Maternal temperature 101.2 (your answer) C. Early decelerations in the fetal heart rate. D. Fetal heart rate baseline 140-160 bpm. Explanation Contractions every 1 ½ minutes lasting 70-80 seconds, is indicative of hyperstimulation of the uterus, which could result in injury to the mother and the fetus if Pitocin is not discontinued.

Correct Q.31)

60% answered this correctly. 40% answered wrong

Myrna a public health nurse knows that to determine possible sources of sexually transmitted infections, the BEST method that may be undertaken is: A. Contact tracing (your answer) B. Community survey C. Mass screening tests D. Interview of suspects Explanation Contact tracing is the most practical and reliable method of finding possible sources of person-to-person transmitted infections, such as sexually transmitted diseases.

Correct Q.32)

55% answered this correctly. 45% answered wrong

When assessing a newborn diagnosed with ductus arteriosus, Nurse Olivia should expect that the child most likely would have an: A. Loud, machinery-like murmur. (your answer) B. Bluish color to the lips. C. Decreased BP reading in the upper extremities D. Increased BP reading in the upper extremities. Explanation A loud, machinery-like murmur is a characteristic finding associated with patent ductus arteriosus.

Incorrect Q.33)

40% answered this correctly. 60% answered wrong

Nurse Reese is reviewing the record of a pregnant client for her first prenatal visit. Which of the following data, if noted on the client‟s record, would alert the nurse that the client is at risk for a spontaneous abortion? A. Age 36 years (your answer) B. History of syphilis (correct answer)

C. History of genital herpes D. History of diabetes mellitus Explanation Maternal infections such as syphilis, toxoplasmosis, and rubella are causes of spontaneous abortion. Correct Q.34)

69% answered this correctly. 31% answered wrong

A 33-year old female client came for consultation at the health center with the chief complaint of fever for a week. Accompanying symptoms were muscle pains and body malaise. A week after the start of fever, the client noted yellowish discoloration of his sclera. History showed that he waded in flood waters about 2 weeks before the onset of symptoms. Based on her history, which disease condition will you suspect? A. Hepatitis A B. Hepatitis B C. Tetanus D. Leptospirosis (your answer) Explanation Leptospirosis is transmitted through contact with the skin or mucous membrane with water or moist soil contaminated with urine of infected animals, like rats.

Correct Q.35)

60% answered this correctly. 40% answered wrong

Nurse Lynette is working in the triage area of an emergency department. She sees that several pediatric clients arrive simultaneously. The client who needs to be treated first is: A. A crying 5 year old child with a laceration on his scalp. B. A 4 year old child with a barking coughs and flushed appearance. C. A 3 year old child with Down syndrome who is pale and asleep in his mother‟s arms. D. A 2 year old infant with stridorous breath sounds, sitting up in his mother‟s arms and drooling. (your answer) Explanation The infant with the airway emergency should be treated first, because of the risk of epiglottitis.

Correct Q.36)

65% answered this correctly. 35% answered wrong

Marie brought her 10 month old infant for consultation because of fever, started 4 days prior to consultation. In determining malaria risk, what will you do? A. Perform a tourniquet test. B. Ask where the family resides. (your answer) C. Get a specimen for blood smear. D. Ask if the fever is present everyday. Explanation Because malaria is endemic, the first question to determine malaria risk is where the client‟s family resides. If the area of residence is not a known endemic area, ask if the child had traveled within the past 6 months, where she was brought and whether she stayed overnight in that area.

Incorrect Q.37)

32% answered this correctly. 68% answered wrong

Before adding potassium to an infant‟s I.V. line, Nurse Ron must be sure to assess whether this infant has: A. Stable blood pressure B. Patant fontanelles C. Moro‟s reflex (your answer) D. Voided (correct answer) Explanation Before administering potassium I.V. to any client, the nurse must first check that the client‟s kidneys are functioning and that the client is voiding. If the client is not voiding, the nurse should withhold the potassium and notify the physician.

Incorrect Q.38)

31% answered this correctly. 69% answered wrong

In Integrated Management of Childhood Illness, the nurse is aware that the severe conditions generally require urgent referral to a hospital. Which of the following severe conditions DOES NOT always require urgent referral to a hospital? A. Mastoiditis (your answer) B. Severe dehydration (correct answer) C. Severe pneumonia D. Severe febrile disease Explanation

The order of priority in the management of severe dehydration is as follows: intravenous fluid therapy, referral to a facility where IV fluids can be initiated within 30 minutes, Oresol or nasogastric tube. When the foregoing measures are not possible or effective, then urgent referral to the hospital is done. Incorrect Q.39)

46% answered this correctly. 54% answered wrong

Vangie is a new B.S.N. graduate. She wants to become a Public Health Nurse. Where should she apply? A. Department of Health B. Provincial Health Office (your answer) C. Regional Health Office D. Rural Health Unit (correct answer) Explanation R.A. 7160 devolved basic health services to local government units (LGU‟s ). The public health nurse is an employee of the LGU.

Correct Q.40)

69% answered this correctly. 31% answered wrong

Tyra experienced painless vaginal bleeding has just been diagnosed as having a placenta previa. Which of the following procedures is usually performed to diagnose placenta previa? A. Amniocentesis B. Digital or speculum examination C. External fetal monitoring D. Ultrasound (your answer) Explanation Once the mother and the fetus are stabilized, ultrasound evaluation of the placenta should be done to determine the cause of the bleeding. Amniocentesis is contraindicated in placenta previa. A digital or speculum examination shouldn‟t be done as this may lead to severe bleeding or hemorrhage. External fetal monitoring won‟t detect a placenta previa, although it will detect fetal distress, which may result from blood loss or placenta separation.

Incorrect Q.41)

32% answered this correctly. 68% answered wrong

Baby Jenny who is small-for-gestation is at increased risk during the transitional period for which complication? A. Anemia probably due to chronic fetal hyposia B. Hyperthermia due to decreased glycogen stores (your answer) C. Hyperglycemia due to decreased glycogen stores D. Polycythemia probably due to chronic fetal hypoxia (correct answer) Explanation The small-for-gestation neonate is at risk for developing polycythemia during the transitional period in an attempt to decrease hypoxia. The neonates are also at increased risk for developing hypoglycemia and hypothermia due to decreased glycogen stores.

Correct Q.42)

62% answered this correctly. 38% answered wrong

A trial for vaginal delivery after an earlier caesareans, would likely to be given to a gravida, who had: A. First low transverse cesarean was for active herpes type 2 infections; vaginal culture at 39 weeks pregnancy was positive. B. First and second caesareans were for cephalopelvic disproportion. C. First caesarean through a classic incision as a result of severe fetal distress. D. First low transverse caesarean was for breech position. Fetus in this pregnancy is in a vertex presentation. (your answer) Explanation This type of client has no obstetrical indication for a caesarean section as she did with her first caesarean delivery.

Correct Q.43)

75% answered this correctly. 25% answered wrong

When teaching umbilical cord care for Jennifer a new mother, the nurse Jenny would include which information? A. Apply peroxide to the cord with each diaper change B. Cover the cord with petroleum jelly after bathing C. Keep the cord dry and open to air (your answer) D. Wash the cord with soap and water each day during a tub bath. Explanation Keeping the cord dry and open to air helps reduce infection and hastens drying. Infants aren‟t given tub bath but are sponged off until the cord falls off. Petroleum jelly prevents the cord from drying and encourages infection. Peroxide could be painful and isn‟t recommended.

Correct

87% answered this correctly. 13% answered wrong

Q.44)

Which of the following drugs is the antidote for magnesium toxicity? A. Calcium gluconate (Kalcinate) (your answer) B. Hydralazine (Apresoline) C. Naloxone (Narcan) D. Rho (D) immune globulin (RhoGAM) Explanation Calcium gluconate is the antidote for magnesium toxicity. Ten milliliters of 10% calcium gluconate is given L.V. push over 3 to 5 minutes. Hydralazine is given for sustained elevated blood pressure in preeclamptic clients. Rho (D) immune globulin is given to women with Rh-negative blood to prevent antibody formation from RH-positive conceptions. Naloxone is used to correct narcotic toxicity. 43% answered this correctly. 57% answered wrong

Correct Q.45)

When the nurse determines whether resources were maximized in implementing Ligtas Tigdas, she is evaluating A. Effectiveness B. Efficiency (your answer) C. Adequacy D. Appropriateness Explanation Efficiency is determining whether the goals were attained at the least possible cost.

Incorrect Q.46)

51% answered this correctly. 49% answered wrong

Which of the following is the most prominent feature of public health nursing? A. It involves providing home care to sick people who are not confined in the hospital. B. Services are provided free of charge to people within the catchments area. C. The public health nurse functions as part of a team providing a public health nursing services. (your answer) D. Public health nursing focuses on preventive, not curative, services. (correct answer) Explanation The catchments area in PHN consists of a residential community, many of whom are well individuals who have greater need for preventive rather than curative services.

Incorrect Q.47)

61% answered this correctly. 39% answered wrong

Nurse Gina is aware that the most common condition found during the second-trimester of pregnancy is: A. Metabolic alkalosis (your answer) B. Respiratory acidosis C. Mastitis D. Physiologic anemia (correct answer) Explanation Hemoglobin values and hematocrit decrease during pregnancy as the increase in plasma volume exceeds the increase in red blood cell production. 76% answered this correctly. 24% answered wrong

Correct Q.48)

Gina is using Oresol in the management of diarrhea of her 3-year old child. She asked you what to do if her child vomits. As a nurse you will tell her to: A. Bring the child to the nearest hospital for further assessment. B. Bring the child to the health center for intravenous fluid therapy. C. Bring the child to the health center for assessment by the physician. D. Let the child rest for 10 minutes then continue giving Oresol more slowly. (your answer) Explanation If the child vomits persistently, that is, he vomits everything that he takes in, he has to be referred urgently to a hospital. Otherwise, vomiting is managed by letting the child rest for 10 minutes and then continuing with Oresol administration. Teach the mother to give Oresol more slowly.

Incorrect Q.49)

44% answered this correctly. 56% answered wrong

The uterus returns to the pelvic cavity in which of the following time frames? A. 7th to 9th day postpartum. (correct answer) B. 2 weeks postpartum.

C. End of 6th week postpartum. (your answer) D. When the lochia changes to alba. Explanation The normal involutional process returns the uterus to the pelvic cavity in 7 to 9 days. A significant involutional complication is the failure of the uterus to return to the pelvic cavity within the prescribed time period. This is known as subinvolution. 55% answered this correctly. 45% answered wrong

Correct Q.50)

May arrives at the health care clinic and tells the nurse that her last menstrual period was 9 weeks ago. She also tells the nurse that a home pregnancy test was positive but she began to have mild cramps and is now having moderate vaginal bleeding. During the physical examination of the client, the nurse notes that May has a dilated cervix. The nurse determines that May is experiencing which type of abortion? A. Inevitable (your answer) B. Incomplete C. Threatened D. Septic Explanation An inevitable abortion is termination of pregnancy that cannot be prevented. Moderate to severe bleeding with mild cramping and cervical dilation would be noted in this type of abortion. 70% answered this correctly. 30% answered wrong

Correct Q.51)

Marlyn is screened for tuberculosis during her first prenatal visit. An intradermal injection of purified protein derivative (PPD) of the tuberculin bacilli is given. She is considered to have a positive test for which of the following results? A. An indurated wheal under 10 mm in diameter appears in 6 to 12 hours. B. An indurated wheal over 10 mm in diameter appears in 48 to 72 hours. (your answer) C. A flat circumcised area under 10 mm in diameter appears in 6 to 12 hours. D. A flat circumcised area over 10 mm in diameter appears in 48 to 72 hours. Explanation A positive PPD result would be an indurated wheal over 10 mm in diameter that appears in 48 to 72 hours. The area must be a raised wheal, not a flat circumcised area to be considered positive. 41% answered this correctly. 59% answered wrong

Incorrect Q.52)

Nurse Ron is aware that unused BCG should be discarded after how many hours of reconstitution? A. 2 hours B. 4 hours (correct answer) C. 8 hours (your answer) D. At the end of the day Explanation While the unused portion of other biologicals in EPI may be given until the end of the day, only BCG is discarded 4 hours after reconstitution. This is why BCG immunization is scheduled only in the morning. 50% answered this correctly. 50% answered wrong

Correct Q.53)

The reason nurse May keeps the neonate in a neutral thermal environment is that when a newborn becomes too cool, the neonate requires: A. Less oxygen, and the newborn‟s metabolic rate increases. B. More oxygen, and the newborn‟s metabolic rate decreases. C. More oxygen, and the newborn‟s metabolic rate increases.(your answer) D. Less oxygen, and the newborn‟s metabolic rate decreases. Explanation When cold, the infant requires more oxygen and there is an increase in metabolic rate. Non-shievering thermogenesis is a complex process that increases the metabolic rate and rate of oxygen consumption, therefore, the newborn increase heat production. 13% answered this correctly. 87% answered wrong

Incorrect Q.54)

Which finding might be seen in baby James a neonate suspected of having an infection? A. B. C. D.

Flushed cheeks Increased temperature (your answer) Decreased temperature (correct answer) Increased activity level

Explanation Temperature instability, especially when it results in a low temperature in the neonate, may be a sign of infection. The neonate‟s color often changes with an infection process but generally becomes ashen or mottled. The neonate with an infection will usually show a decrease in activity level or lethargy. Incorrect Q.55)

73% answered this correctly. 27% answered wrong

During vaginal examination of Janah who is in labor, the presenting part is at station plus two. Nurse, correctly interprets it as: A. Presenting part is 2 cm above the plane of the ischial spines.(your answer) B. Biparietal diameter is at the level of the ischial spines. C. Presenting part in 2 cm below the plane of the ischial spines.(correct answer) D. Biparietal diameter is 2 cm above the ischial spines. Explanation Fetus at station plus two indicates that the presenting part is 2 cm below the plane of the ischial spines.

Incorrect Q.56)

21% answered this correctly. 79% answered wrong

Following a precipitous delivery, examination of the client's vagina reveals a fourth-degree laceration. Which of the following would be contraindicated when caring for this client? A. Applying cold to limit edema during the first 12 to 24 hours. B. Instructing the client to use two or more peripads to cushion the area. (correct answer) C. Instructing the client on the use of sitz baths if ordered. D. Instructing the client about the importance of perineal (kegel) exercises. (your answer) Explanation Using two or more peripads would do little to reduce the pain or promote perineal healing. Cold applications, sitz baths, and Kegel exercises are important measures when the client has a fourth-degree laceration.

Incorrect Q.57)

29% answered this correctly. 71% answered wrong

Which of the following is normal newborn calorie intake? A. 110 to 130 calories per kg. (correct answer) B. 30 to 40 calories per lb of body weight. C. At least 2 ml per feeding D. 90 to 100 calories per kg (your answer) Explanation Calories per kg is the accepted way of determined appropriate nutritional intake for a newborn. The recommended calorie requirement is 110 to 130 calories per kg of newborn body weight. This level will maintain a consistent blood glucose level and provide enough calories for continued growth and development.

Correct Q.58)

64% answered this correctly. 36% answered wrong

Susie brought her 4 years old daughter to the RHU because of cough and colds. Following the IMCI assessment guide, which of the following is a danger sign that indicates the need for urgent referral to a hospital? A. Inability to drink (your answer) B. High grade fever C. Signs of severe dehydration D. Cough for more than 30 days Explanation A sick child aged 2 months to 5 years must be referred urgently to a hospital if he/she has one or more of the following signs: not able to feed or drink, vomits everything, convulsions, abnormally sleepy or difficult to awaken.

Correct Q.59)

48% answered this correctly. 52% answered wrong

How should Nurse Michelle guide a child who is blind to walk to the playroom? A. Without touching the child, talk continuously as the child walks down the hall. B. Walk one step ahead, with the child‟s hand on the nurse‟s elbow.(your answer) C. Walk slightly behind, gently guiding the child forward. D. Walk next to the child, holding the child‟s hand. Explanation This procedure is generally recommended to follow in guiding a person who is blind.

Incorrect Q.60)

28% answered this correctly. 72% answered wrong

Barangay Pinoy had an outbreak of German measles. To prevent congenital rubella, what is the BEST advice that you can give to

women in the first trimester of pregnancy in the barangay Pinoy? A. Advice them on the signs of German measles. B. Avoid crowded places, such as markets and movie houses.(your answer) C. Consult at the health center where rubella vaccine may be given. D. Consult a physician who may give them rubella immunoglobulin.(correct answer) Explanation Rubella vaccine is made up of attenuated German measles viruses. This is contraindicated in pregnancy. Immune globulin, a specific prophylactic against German measles, may be given to pregnant women. Correct Q.61)

61% answered this correctly. 39% answered wrong

Chris a 4-month old infant was brought by her mother to the health center because of cough. His respiratory rate is 42/minute. Using the Integrated Management of Child Illness (IMCI) guidelines of assessment, his breathing is considered as: A. Fast B. Slow C. Normal (your answer) D. Insignificant Explanation In IMCI, a respiratory rate of 50/minute or more is fast breathing for an infant aged 2 to 12 months.

Correct Q.62)

80% answered this correctly. 20% answered wrong

A fullterm client is in labor. Nurse Betty is aware that the fetal heart rate would be: A. 80 to 100 beats/minute B. 100 to 120 beats/minute C. 120 to 160 beats/minute (your answer) D. 160 to 180 beats/minute Explanation A rate of 120 to 160 beats/minute in the fetal heart appropriate for filling the heart with blood and pumping it out to the system.

Incorrect Q.63)

64% answered this correctly. 36% answered wrong

Tony is aware the Chairman of the Municipal Health Board is: A. Mayor (correct answer) B. Municipal Health Officer C. Public Health Nurse D. Any qualified physician (your answer) Explanation The local executive serves as the chairman of the Municipal Health Board.

Correct Q.64)

60% answered this correctly. 40% answered wrong

Nurse Ryan is aware that the best initial approach when trying to take a crying toddler‟s temperature is: A. Talk to the mother first and then to the toddler. (your answer) B. Bring extra help so it can be done quickly. C. Encourage the mother to hold the child. D. Ignore the crying and screaming. Explanation When dealing with a crying toddler, the best approach is to talk to the mother and ignore the toddler first. This approach helps the toddler get used to the nurse before she attempts any procedures. It also gives the toddler an opportunity to see that the mother trusts the nurse.

Correct Q.65)

34% answered this correctly. 66% answered wrong

Rh isoimmunization in a pregnant client develops during which of the following conditions? A. Rh-positive maternal blood crosses into fetal blood, stimulating fetal antibodies. B. Rh-positive fetal blood crosses into maternal blood, stimulating maternal antibodies. (your answer) C. Rh-negative fetal blood crosses into maternal blood, stimulating maternal antibodies. D. Rh-negative maternal blood crosses into fetal blood, stimulating fetal antibodies. Explanation Rh isoimmunization occurs when Rh-positive fetal blood cells cross into the maternal circulation and stimulate maternal antibody

production. In subsequent pregnancies with Rh-positive fetuses, maternal antibodies may cross back into the fetal circulation and destroy the fetal blood cells. Incorrect Q.66)

51% answered this correctly. 49% answered wrong

Emily has gestational diabetes and it is usually managed by which of the following therapy? A. Diet (correct answer) B. Long-acting insulin (your answer) C. Oral hypoglycemic D. Oral hypoglycemic drug and insulin Explanation Clients with gestational diabetes are usually managed by diet alone to control their glucose intolerance. Oral hypoglycemic drugs are contraindicated in pregnancy. Long-acting insulin usually isn‟t needed for blood glucose control in the client with gestational diabetes.

Incorrect Q.67)

29% answered this correctly. 71% answered wrong

Jimmy a 2-year old child revealed “baggy pants”. As a nurse, using the IMCI guidelines, how will you manage Jimmy? A. Refer the child urgently to a hospital for confinement. (correct answer) B. Coordinate with the social worker to enroll the child in a feeding program. C. Make a teaching plan for the mother, focusing on menu planning for her child. D. Assess and treat the child for health problems like infections and intestinal parasitism. (your answer) Explanation “Baggy pants” is a sign of severe marasmus. The best management is urgent referral to a hospital.

Incorrect Q.68)

32% answered this correctly. 68% answered wrong

Nurse John is knowledgeable that usually individual twins will grow appropriately and at the same rate as singletons until how many weeks? A. 16 to 18 weeks B. 18 to 22 weeks (your answer) C. 30 to 32 weeks (correct answer) D. 38 to 40 weeks Explanation Individual twins usually grow at the same rate as singletons until 30 to 32 weeks‟ gestation, then twins don‟t‟ gain weight as rapidly as singletons of the same gestational age. The placenta can no longer keep pace with the nutritional requirements of both fetuses after 32 weeks, so there‟s some growth retardation in twins if they remain in utero at 38 to 40 weeks.

Correct Q.69)

64% answered this correctly. 36% answered wrong

Maureen, a primigravida client, age 20, has just completed a difficult, forceps-assisted delivery of twins. Her labor was unusually long and required oxytocin (Pitocin) augmentation. The nurse who's caring for her should stay alert for: A. Uterine inversion B. Uterine atony (your answer) C. Uterine involution D. Uterine discomfort Explanation Multiple fetuses, extended labor stimulation with oxytocin, and traumatic delivery commonly are associated with uterine atony, which may lead to postpartum hemorrhage. Uterine inversion may precede or follow delivery and commonly results from apparent excessive traction on the umbilical cord and attempts to deliver the placenta manually. Uterine involution and some uterine discomfort are normal after delivery.

Correct Q.70)

71% answered this correctly. 29% answered wrong

Nurse Michelle is assessing a 24 year old client with a diagnosis of hydatidiform mole. She is aware that one of the following is unassociated with this condition? A. Excessive fetal activity. (your answer) B. Larger than normal uterus for gestational age. C. Vaginal bleeding D. Elevated levels of human chorionic gonadotropin. Explanation The most common signs and symptoms of hydatidiform mole includes elevated levels of human chorionic gonadotropin, vaginal bleeding, larger than normal uterus for gestational age, failure to detect fetal heart activity even with sensitive instruments, excessive nausea and vomiting, and early development of pregnancy-induced hypertension. Fetal activity would not be noted.

Correct Q.71)

62% answered this correctly. 38% answered wrong

A pregnant client is receiving magnesium sulfate for severe pregnancy induced hypertension (PIH). The clinical findings that would warrant use of the antidote , calcium gluconate is: A. Urinary output 90 cc in 2 hours. B. Absent patellar reflexes. (your answer) C. Rapid respiratory rate above 40/min. D. Rapid rise in blood pressure. Explanation Absence of patellar reflexes is an indicator of hypermagnesemia, which requires administration of calcium gluconate.

Incorrect Q.72)

39% answered this correctly. 61% answered wrong

The nurse is caring for a primigravid client in the labor and delivery area. Which condition would place the client at risk for disseminated intravascular coagulation (DIC)? A. Intrauterine fetal death. (correct answer) B. Placenta accreta. C. Dysfunctional labor. (your answer) D. Premature rupture of the membranes. Explanation Intrauterine fetal death, abruptio placentae, septic shock, and amniotic fluid embolism may trigger normal clotting mechanisms; if clotting factors are depleted, DIC may occur. Placenta accreta, dysfunctional labor, and premature rupture of the membranes aren't associated with DIC.

Correct Q.73)

35% answered this correctly. 65% answered wrong

Celeste who used heroin during her pregnancy delivers a neonate. When assessing the neonate, the nurse Lhynnette expects to find: A. Lethargy 2 days after birth. B. Irritability and poor sucking. (your answer) C. A flattened nose, small eyes, and thin lips. D. Congenital defects such as limb anomalies. Explanation Neonates of heroin-addicted mothers are physically dependent on the drug and experience withdrawal when the drug is no longer supplied. Signs of heroin withdrawal include irritability, poor sucking, and restlessness. Lethargy isn't associated with neonatal heroin addiction. A flattened nose, small eyes, and thin lips are seen in infants with fetal alcohol syndrome. Heroin use during pregnancy hasn't been linked to specific congenital anomalies.

Incorrect Q.74)

50% answered this correctly. 50% answered wrong

A 23 year old client is having her menstrual period every 2 weeks that last for 1 week. This type of menstrual pattern is bets defined by: A. Menorrhagia (correct answer) B. Metrorrhagia (your answer) C. Dyspareunia D. Amenorrhea Explanation Menorrhagia is an excessive menstrual period.

Incorrect Q.75)

25% answered this correctly. 75% answered wrong

The most prevalent form of meningitis among children aged 2 months to 3 years is caused by which microorganism? A. Hemophilus influenzae (correct answer) B. Morbillivirus C. Steptococcus pneumoniae D. Neisseria meningitidis (your answer) Explanation Hemophilus meningitis is unusual over the age of 5 years. In developing countries, the peak incidence is in children less than 6 months of age. Morbillivirus is the etiology of measles. Streptococcus pneumoniae and Neisseria meningitidis may cause meningitis, but age distribution is not specific in young children.

Correct

68% answered this correctly. 32% answered wrong

Q.76)

Mickey a 3-year old client was brought to the health center with the chief complaint of severe diarrhea and the passage of “rice water” stools. The client is most probably suffering from which condition? A. Giardiasis B. Cholera (your answer) C. Amebiasis D. Dysentery Explanation Passage of profuse watery stools is the major symptom of cholera. Both amebic and bacillary dysentery are characterized by the presence of blood and/or mucus in the stools. Giardiasis is characterized by fat malabsorption and, therefore, steatorrhea.

Incorrect Q.77)

42% answered this correctly. 58% answered wrong

Nurse Arnold knows that the following changes in respiratory functioning during pregnancy is considered normal: A. Increased tidal volume (correct answer) B. Increased expiratory volume C. Decreased inspiratory capacity (your answer) D. Decreased oxygen consumption Explanation A pregnant client breathes deeper, which increases the tidal volume of gas moved in and out of the respiratory tract with each breath. The expiratory volume and residual volume decrease as the pregnancy progresses. The inspiratory capacity increases during pregnancy. The increased oxygen consumption in the pregnant client is 15% to 20% greater than in the nonpregnant state.

Incorrect Q.78)

45% answered this correctly. 55% answered wrong

Hypoxia is a common complication of laryngotracheobronchitis. Nurse Oliver should frequently assess a child with laryngotracheobronchitis for: A. Drooling B. Muffled voice (your answer) C. Restlessness (correct answer) D. Low-grade fever Explanation In a child, restlessness is the earliest sign of hypoxia. Late signs of hypoxia in a child are associated with a change in color, such as pallor or cyanosis.

Incorrect Q.79)

42% answered this correctly. 58% answered wrong

During tube feeding, how far above an infant‟s stomach should the nurse hold the syringe with formula? A. 6 inches (correct answer) B. 12 inches (your answer) C. 18 inches D. 24 inches Explanation This distance allows for easy flow of the formula by gravity, but the flow will be slow enough not to overload the stomach too rapidly.

Correct Q.80)

74% answered this correctly. 26% answered wrong

Which action should nurse Marian include in the care plan for a 2 month old with heart failure? A. Feed the infant when he cries. B. Allow the infant to rest before feeding. (your answer) C. Bathe the infant and administer medications before feeding. D. Weigh and bathe the infant before feeding. Explanation Because feeding requires so much energy, an infant with heart failure should rest before feeding.

Correct Q.81)

41% answered this correctly. 59% answered wrong

A baby girl is born 8 weeks premature. At birth, she has no spontaneous respirations but is successfully resuscitated. Within several hours she develops respiratory grunting, cyanosis, tachypnea, nasal flaring, and retractions. She's diagnosed with respiratory distress syndrome, intubated, and placed on a ventilator. Which nursing action should be included in the baby's plan of care to prevent retinopathy of prematurity? A. Cover his eyes while receiving oxygen.

B. Keep her body temperature low. C. Monitor partial pressure of oxygen (Pao2) levels. (your answer) D. Humidify the oxygen. Explanation Monitoring PaO2 levels and reducing the oxygen concentration to keep PaO2 within normal limits reduces the risk of retinopathy of prematurity in a premature infant receiving oxygen. Covering the infant's eyes and humidifying the oxygen don't reduce the risk of retinopathy of prematurity. Because cooling increases the risk of acidosis, the infant should be kept warm so that his respiratory distress isn't aggravated. Correct Q.82)

74% answered this correctly. 26% answered wrong

The student nurse is aware that the pathognomonic sign of measles is Koplik‟s spot and you may see Koplik‟s spot by inspecting the: A. Nasal mucosa B. Buccal mucosa (your answer) C. Skin on the abdomen D. Skin on neck Explanation Koplik‟s spot may be seen on the mucosa of the mouth or the throat.

Correct Q.83)

62% answered this correctly. 38% answered wrong

Magnesium sulfate is given to Jemma with preeclampsia to prevent which of the following condition? A. Hemorrhage B. Hypertension C. Hypomagnesemia D. Seizure (your answer) Explanation The anticonvulsant mechanism of magnesium is believes to depress seizure foci in the brain and peripheral neuromuscular blockade. Hypomagnesemia isn‟t a complication of preeclampsia. Antihypertensive drug other than magnesium are preferred for sustained hypertension. Magnesium doesn‟t help prevent hemorrhage in preeclamptic clients.

Incorrect Q.84)

37% answered this correctly. 63% answered wrong

Jannah is admitted to the labor and delivery unit. The critical laboratory result for this client would be: A. Oxygen saturation B. Iron binding capacity C. Blood typing (correct answer) D. Serum Calcium (your answer) Explanation Blood type would be a critical value to have because the risk of blood loss is always a potential complication during the labor and delivery process. Approximately 40% of a woman‟s cardiac output is delivered to the uterus, therefore, blood loss can occur quite rapidly in the event of uncontrolled bleeding.

Incorrect Q.85)

56% answered this correctly. 44% answered wrong

Cammile with sickle cell anemia has an increased risk for having a sickle cell crisis during pregnancy. Aggressive management of a sickle cell crisis includes which of the following measures? A. Antihypertensive agents B. Diuretic agents (your answer) C. I.V. fluids (correct answer) D. Acetaminophen (Tylenol) for pain Explanation A sickle cell crisis during pregnancy is usually managed by exchange transfusion oxygen, and L.V. Fluids. The client usually needs a stronger analgesic than acetaminophen to control the pain of a crisis. Antihypertensive drugs usually aren‟t necessary. Diuretic wouldn‟t be used unless fluid overload resulted.

Correct Q.86)

64% answered this correctly. 36% answered wrong

After reviewing the Myrna‟s maternal history of magnesium sulfate during labor, which condition would nurse Richard anticipate as a potential problem in the neonate? A. Hypoglycemia B. Jitteriness

C. Respiratory depression (your answer) D. Tachycardia Explanation Magnesium sulfate crosses the placenta and adverse neonatal effects are respiratory depression, hypotonia, and bradycardia. The serum blood sugar isn‟t affected by magnesium sulfate. The neonate would be floppy, not jittery. Correct Q.87)

78% answered this correctly. 22% answered wrong

A pregnant woman accompanied by her husband, seeks admission to the labor and delivery area. She states that she's in labor and says she attended the facility clinic for prenatal care. Which question should the nurse Oliver ask her first? A. “Do you have any chronic illnesses?” B. “Do you have any allergies?” C. “What is your expected due date?” (your answer) D. “Who will be with you during labor?” Explanation When obtaining the history of a client who may be in labor, the nurse's highest priority is to determine her current status, particularly her due date, gravidity, and parity. Gravidity and parity affect the duration of labor and the potential for labor complications. Later, the nurse should ask about chronic illnesses, allergies, and support persons.

Correct Q.88)

55% answered this correctly. 45% answered wrong

Nurse Ron is aware that the gestational age of a conceptus that is considered viable (able to live outside the womb) is: A. 8 weeks B. 12 weeks C. 24 weeks (your answer) D. 32 weeks Explanation At approximately 23 to 24 weeks‟ gestation, the lungs are developed enough to sometimes maintain extrauterine life. The lungs are the most immature system during the gestation period. Medical care for premature labor begins much earlier (aggressively at 21 weeks‟ gestation)

Correct Q.89)

29% answered this correctly. 71% answered wrong

Dr. Esteves decides to artificially rupture the membranes of a mother who is on labor. Following this procedure, the nurse Hazel checks the fetal heart tones for which the following reasons? A. To determine fetal well-being. B. To assess for prolapsed cord (your answer) C. To assess fetal position D. To prepare for an imminent delivery. Explanation After a client has an amniotomy, the nurse should assure that the cord isn't prolapsed and that the baby tolerated the procedure well. The most effective way to do this is to check the fetal heart rate. Fetal well-being is assessed via a nonstress test. Fetal position is determined by vaginal examination. Artificial rupture of membranes doesn't indicate an imminent delivery.

Correct Q.90)

57% answered this correctly. 43% answered wrong

The community nurse is aware that the biological used in Expanded Program on Immunization (EPI) should NOT be stored in the freezer? A. DPT (your answer) B. Oral polio vaccine C. Measles vaccine D. MMR Explanation DPT is sensitive to freezing. The appropriate storage temperature of DPT is 2 to 8° C only. OPV and measles vaccine are highly sensitive to heat and require freezing. MMR is not an immunization in the Expanded Program on Immunization.

Incorrect Q.91)

36% answered this correctly. 64% answered wrong

Which of the following classifications applies to monozygotic twins for whom the cleavage of the fertilized ovum occurs more than 13 days after fertilization? A. conjoined twins (correct answer) B. diamniotic dichorionic twins C. diamniotic monochorionic twin (your answer)

D. monoamniotic monochorionic twins Explanation The type of placenta that develops in monozygotic twins depends on the time at which cleavage of the ovum occurs. Cleavage in conjoined twins occurs more than 13 days after fertilization. Cleavage that occurs less than 3 day after fertilization results in diamniotic dicchorionic twins. Cleavage that occurs between days 3 and 8 results in diamniotic monochorionic twins. Cleavage that occurs between days 8 to 13 result in monoamniotic monochorionic twins. 42% answered this correctly. 58% answered wrong

Correct Q.92)

Marjorie has just given birth at 42 weeks‟ gestation. When the nurse assessing the neonate, which physical finding is expected? A. A sleepy, lethargic baby B. Lanugo covering the body C. Desquamation of the epidermis (your answer) D. Vernix caseosa covering the body Explanation Postdate fetuses lose the vernix caseosa, and the epidermis may become desquamated. These neonates are usually very alert. Lanugo is missing in the postdate neonate. 67% answered this correctly. 33% answered wrong

Correct Q.93)

A neonate begins to gag and turns a dusky color. What should the nurse do first? A. Calm the neonate. B. Notify the physician. C. Provide oxygen via face mask as ordered D. Aspirate the neonate‟s nose and mouth with a bulb syringe.(your answer) Explanation The nurse's first action should be to clear the neonate's airway with a bulb syringe. After the airway is clear and the neonate's color improves, the nurse should comfort and calm the neonate. If the problem recurs or the neonate's color doesn't improve readily, the nurse should notify the physician. Administering oxygen when the airway isn't clear would be ineffective. 60% answered this correctly. 40% answered wrong

Correct Q.94)

To promote comfort during labor, the nurse John advises a client to assume certain positions and avoid others. Which position may cause maternal hypotension and fetal hypoxia? A. Lateral position B. Squatting position C. Supine position (your answer) D. Standing position Explanation The supine position causes compression of the client's aorta and inferior vena cava by the fetus. This, in turn, inhibits maternal circulation, leading to maternal hypotension and, ultimately, fetal hypoxia. The other positions promote comfort and aid labor progress. For instance, the lateral, or side-lying, position improves maternal and fetal circulation, enhances comfort, increases maternal relaxation, reduces muscle tension, and eliminates pressure points. The squatting position promotes comfort by taking advantage of gravity. The standing position also takes advantage of gravity and aligns the fetus with the pelvic angle. 52% answered this correctly. 48% answered wrong

Correct Q.95)

In doing a child‟s admission assessment, Nurse Betty should be alert to note which signs or symptoms of chronic lead poisoning? A. Irritability and seizures (your answer) B. Dehydration and diarrhea C. Bradycardia and hypotension D. Petechiae and hematuria Explanation Lead poisoning primarily affects the CNS, causing increased intracranial pressure. This condition results in irritability and changes in level of consciousness, as well as seizure disorders, hyperactivity, and learning disabilities. 80% answered this correctly. 20% answered wrong

Correct Q.96)

Angel was diagnosed as having Dengue fever. You will say that there is slow capillary refill when the color of the nailbed that you pressed does not return within how many seconds? A. B. C. D.

3 seconds (your answer) 6 seconds 9 seconds 10 seconds

Explanation Adequate blood supply to the area allows the return of the color of the nailbed within 3 seconds. Correct Q.97)

54% answered this correctly. 46% answered wrong

To evaluate a woman‟s understanding about the use of diaphragm for family planning, Nurse Trish asks her to explain how she will use the appliance. Which response indicates a need for further health teaching? A. “I should check the diaphragm carefully for holes every time I use it” B. “I may need a different size of diaphragm if I gain or lose weight more than 20 pounds” C. “The diaphragm must be left in place for atleast 6 hours after intercourse” D. “I really need to use the diaphragm and jelly most during the middle of my menstrual cycle”. (your answer) Explanation The woman must understand that, although the “fertile” period is approximately mid-cycle, hormonal variations do occur and can result in early or late ovulation. To be effective, the diaphragm should be inserted before every intercourse.

Correct Q.98)

62% answered this correctly. 38% answered wrong

Nurse Carla should know that the most common causative factor of dermatitis in infants and younger children is: A. Baby oil B. Baby lotion C. Laundry detergent (your answer) D. Powder with cornstarch Explanation Eczema or dermatitis is an allergic skin reaction caused by an offending allergen. The topical allergen that is the most common causative factor is laundry detergent.

Incorrect Q.99)

35% answered this correctly. 65% answered wrong

In a mothers‟ class, Nurse Lhynnete discussed childhood diseases such as chicken pox. Which of the following statements about chicken pox is correct? A. The older one gets, the more susceptible he becomes to the complications of chicken pox. (correct answer) B. A single attack of chicken pox will prevent future episodes, including conditions such as shingles. (your answer) C. To prevent an outbreak in the community, quarantine may be imposed by health authorities. D. Chicken pox vaccine is best given when there is an impending outbreak in the community. Explanation Chicken pox is usually more severe in adults than in children. Complications, such as pneumonia, are higher in incidence in adults.

Correct

64% answered this correctly. 36% answered wrong

Q.100) Which symptom would indicate the Baby Alexandra was adapting appropriately to extra-uterine life without difficulty? A. Nasal flaring B. Light audible grunting C. Respiratory rate 40 to 60 breaths/minute (your answer) D. Respiratory rate 60 to 80 breaths/minute Explanation A respiratory rate 40 to 60 breaths/minute is normal for a neonate during the transitional period. Nasal flaring, respiratory rate more than 60 breaths/minute, and audible grunting are signs of respiratory distress.

View more...

Comments

Copyright ©2017 KUPDF Inc.
SUPPORT KUPDF